The energy efficiency of cars


Chart updated 28 February to take account of this comment from Profbaldwin.

The future of motor vehicles lies in improved efficiency and that is to the left of the gasoline ICE in the chart. That future is electric vehicles powered by high ERoEI renewable electricity.

This post is not intended to provide a rigorous analysis of automobile efficiency but aims to provide an overview of the efficiencies of different drive system - fuel combinations.

The procedure followed is to identify the main energy efficiency factors and to multiply these to provide the overall efficiency in %. The efficiency of fuel source is based on ERoEI where:

ERoEI = energy procured / energy used to procure energy
Efficiency = (ERoEI-1)/ERoEI

I have not considered the energy embedded in the vehicles and energy losses downstream from the motor in the drive systems. The objective is to highlight the major differences between the 4 fuel / drive system combinations.

The all electric car powered by renewable wind electricity

ERoEI for wind ~ 20, efficiency factor = 0.95
Grid transmission losses = 0.9
Battery efficiency = 0.97
Motor efficiency = 0.92

Combined efficiency = 76.3%

Gasoline internal combustion engine (ICE)

Procuring oil, ERoEI = 30 (assumed), efficiency = 0.967
Refining and transport losses = 0.9
ICE efficiency = 0.4

Combined efficiency = 34.8%

Hydrogen fuel cell

The calculation is based on producing hydrogen from electrolysis of water using wind power electricity and is based heavily on an analysis by Ulf Bossel (reference at end).

Wind power, as for all electric car = 0.95
Losses due to electrolysis of water = 0.7
Compression of hydrogen = 0.9
Losses during distribution = 0.9
Losses during hydrogen transfer = 0.97
Fuel cell efficiency = 0.5
Motor efficiency = 0.92

Combined efficiency = 24%

Bio-ethanol internal combustion engine

ERoEI for temperate latitude ethanol ~ 1.5, efficiency = 0.33
Processing and delivery (estimate) = 0.95
ICE efficiency = 0.4

Combined efficiency = 12.5%

Hybrids and plug-in hybrids

I've not covered these but assume that their efficiency will lie between gasoline ICE and all electric.

The world, and in particular the UK, is currently in the eye of the storm of a full blown energy crisis. I think virtually everyone is agreed that improving energy efficiency is essential to the short-term survival of our industrial economies. It is therefore extremely pertinent to ask why EU and US governments have and are still supporting ethanol production for vehicular transportation? And in my own country (Scotland) why it is that our parliament continues to support hydrogen fuel cell research and to offer the false hope of a hydrogen economy?

At the same time it is pertinent to ask what on earth has driven Saab and Volvo to produce ethanol powered vehicles that are arguably the least energy efficient cars built in recent decades. What are their engineers thinking about, and why?

The future of motor vehicles lies in improved efficiency and that is to the left of the gasoline ICE in the chart up top. That future is electric vehicles powered by high ERoEI renewable electricity.

On a final more light hearted note, many UK readers will be familiar with the BBC motoring program Top Gear, hosted by Jeremy Clarkson the self styled opponent of all things politically correct who recently added to his notoriety by describing Prime Minister Gordon Brown as a one eyed Scottish Idiot. A number of weeks ago Top Gear road tested the Tesla all electric car to destruction and a Honda hydrogen fuel cell vehicle - which the Top Gear team loved. They explained that hydrogen was abundant, but stuck on to other stuff. And all that had to be done was to scrape it off. Guys, I'm a great fan of your show, but please take some time to ponder the chart up top and to maybe bring some energy reality to the motoring debate.




Top Gear: Perhaps Clarkson would like to test drive this model? Shown is the all electric Fiat 500.

Some data sources:

Hydrogen fuel cells
Internal combustion engine
Electric motors
Batteries
Wind power

Thank you for the analysis, but ultimately how useful is it?

"World Book Encyclopedia. Chicago:, 2001." estimates that there were around 450,000,000 passenger cars in the world. It is expected, all things being equal, that that number will double by 2031.

So, in a world increasingly staring serious CC in the face and, if the economy were to drag itself out of recession (and the collaspsing motor industry actually decided to increase production again) close approaching peak oil, what you are proposing is new ways of burning up more fossil fuels?

You also say: "The world, and in particular the UK, is currently in the eye of the storm of a full blown energy crisis. I think virtually everyone is agreed that improving energy efficiency is essential to the short-term survival of our industrial economies." Well no, I don't agree - I think that structural energy demand reduction is "essential to the short-term survival of our industrial economies." People walking, cycling, catching the bus and taking the train and leaving their cars, petrol, diesel, hydrogen or electric at home! Or if they do still need to travel by car, two people sharing a single current car model will be far more efficient that those two each driving their own electric cars. Taxes on single occupancy could kick start that one, whereas waiting for all these millions of electric cars to sort us out will take .. err, how long?

We need to be using the UK's dwindling resources to insulate and retrofit our homes and public buildings so that our future energy demand is reduced for decades (if not centuries) to come - we all need to live somewhere, but we don't all need a car. The UK is in a mess in terms of indigenous oil and gas supplies and hence our balance of payments is looking increasingly unhealthy as (I seem to remember) you have so excellently pointed out in some of your other posts. To squander these things by replacing the UKs existing cars sounds well intentioned, but I cannot see it as the answer. Remember, you are only talking about changing the engine - how much oil, fossil fuelled electricity and raw materials will go into the paintwork, interior plastics, tyres, cabling, roads, maintenance vehicles and on and on for these "efficient" electric vehicles?

The remark about Top Gear - "I'm a great fan of your show" I think explains it all - you don't want your world to not include private cars. Top Gear makes me scream (not in a good way), and Jeremy Clarkson, the 'speed god' of the show has things like this to say:

"Now we've been told in this new series, we've got to feature more green cars. So here's one. It's really the greenest car we could find, really." (A bright green Lamborghini Murcielago)

"Telling people at a dinner party you drive a Nissan Almera is like telling them you've got the ebola virus and you're about to sneeze."

"Speed has never killed anyone, suddenly becoming stationary... That's what gets you."

"We all know that small cars are good for us. But so is cod liver oil. And jogging."

“I don’t understand bus lanes. Why do poor people have to get to places quicker than I do?”

"I was reading The Mirror the other day and came across a letter from a reader who wrote, 'I was riding my bike to work when this red Ferrari pulled up next to me. Out of the window, Jeremy Clarkson shouted 'Get a car', and drove off.' What I actually said was, 'Get a car you hatchet faced, leaf-eating N**i."

and finally

Clarksons highway code on cyclists: "trespassers in the motorcars domain, they do not pay road tax and therefore have no right to be on the road, some of them even believe they are going fast enough to not be an obstruction. Run them down to prove them wrong."

... so, the alternatives to the car may seem rather drab when you're a "fan" of the above type of thinking, but let's face it, any sustainable future for this world does not include most of us whizzing around in private cars, electric or otherwise.

Euan, there are a couple more losses to include into electric cars:

Battery charger's transformer, rectifier and choke. I doubt a battery charger is more than 90% efficient.
Power electronics within the car. Industrial drives are about 95% at rated output, but this falls to zero as output frequency tends to zero. Those used in electric vehicles will likely behave similarly.

Also:

The efficiency of an electric motor is a curve. At low speed high torque, the motor efficiency will be very low. The ICE vehicle suffers from this as well, but there is a danger to confuse theoretical efficiency with what will be realistically achievable. The grid has to supply energy that takes account of the "real life" efficiency.

It has been regularly quoted that compressing hydrogen to 200 bar that is consumes one third of the energy contained, so you are being generous here. I saw the top gear episode to which you refer, and I was not sure if they understood the implications of "detaching" hydrogen from oxygen or just chose to "sarcastically" ignore it.
Also Euan, please spell Program "Programme"!! You live in the UK after all.

There is no infrastructure to support electric cars, despite what those in fantasy land claim. Tens of thousands of dwellings in the uk have no driveway and their cars are parked on the public highway over night. Unless we drag extension leads over the pavement, there is work to be done here. There are many destinations that will not have charging points as well. There are many challanges to overcome regarding electric vehicles. Unfortunately, many folk like to pretend otherwise.

As a new Prius owner living in Vermont I was reminded the other minus 20deg F day of an overlooked ( by me ) fact about batteries: the AH storage capacity goes down drastically as a function of temperature. At -20C we can use only ~50% of the battery's design capacity of Ampere-Hours of storage ..... oops.... there goes the useful range per charge, which OBTW seems to be one of the most difficult design criteria to satisfy.

Put the model in the pix in furs and maybe some muckluks, for a more-to-the-point illustration of a practical vehicle.

http://www.bdbatteries.com/peukert.php

Electric battery warmers work down to at least -40, draw about 20 watts, available at Canadian Tire stores

doesn't help much when you're on the road at -35F. duh.

or are you going to run it off the battery itself?

on a Prius there's no problem starting on a cold morning ..... it's just that the hybrid-system-battery is just another chemical reaction and decreases capacity with decreasing temp.

In a HEV like the Prius, the battery pack can be kept inside and warmed up as the vehicle warms up, so any concerns about temperature w/r/t NiMH capacity aren't an issue. In the case of a PHEV or EV, the manufacturer would almost certainly use a Lithium based chemistry since those are the cheapest per kWh stored right now given a PHEV/EV app, and have very good low temperature performance.

That depends on chemistry. In your case, the Prius uses NiMH batteries, so a link on the relationship between temperature and capacity for different kinds of lead acid chemistries probably isn't the best place to look, at least in terms of current tech. Anyone can put together their own lead acid EV, but that's probably not what we're going to see in the future outside of the lead acid/capacitor combo, maybe...

"The efficiency of an electric motor is a curve"

True for induction machines but not for modern permanent magnet machines, these exhibit a very high part-load efficiency.

EDIT: Of course a PM machine will be more expensive!

crobar, I agree totally, though copper losses in PM machines will be fixed for a given torque so the efficiency will still fall, but not as much. When I suggested permanent magnet machines would be used (on another post) I got slated for it by Engineer_Poet, You just can't win!

The I sq R law applies to everything electrical including PM motors. I represents amps and R is resistance measured in ohms. Ohms are mostly constant while amps are a function of torque. Therefore a motor's efficiency is drops at the square of load. Light loads are very efficient while heavy loads are only a fraction of peak efficiency. The same law applies to fuel cells which only match their high efficiency claims at very light loads.

You've got that almost entirely wrong.

1) I sq R applies, in synchronous motors, only to that part of the impedance presented by the motors which is due to resistance. Most impedance in synchronous motors occurs due to inductance. Also in typical high-efficiency synchronous motors, the field is created by permanent magnets which make no contribution to energy use. In induction motors, same except must also contend with high currents at low voltages, therefore high I sq R losses in cheap aluminum conductors in rotors, often reduced by using costly copper bars in high-Q induction motors eg. Tesla.

2) A typical electric motor's efficiency INCREASES with the load up to rated output, heavily dependent on issues such as a) synchronous or inductive? b) if inductive, what rotor resistance? c) what tradeoffs has the manufacturer made regarding i) winding design and conductor cross-section ii) what quality of magnetic steel chosen? iii) what cooling method chosen, if air, what fan design etc? iv) what engineering tradeoffs made regarding bearings vs. cost etc?

Most reasonable size electric motors will achieve peak efficiency at full rated power, and effic. will drop off dramatically at light loads.

Lengould,

Just to expand on what you say, (have said this above) maximum efficiency occurs when the "load dependant loss = Non load dependant loss". This can be simplified to saying "copper (I*I*R) = Iron (EDDY+Hysterisis)". This applies to transformers as well, and their will be a mechanical equivalent I expect.

At no load, there are no copper losses on a PM motor if electronically commutated since current is prop. to torque.
An induction motor has to draw magnetising current so it has a "no load" copper loss, though less than at full load. For a given frequency and excitation (flux density) Stator iron losses are fixed and rotor iron losses are often ignored altogether, because the rotor slip frequency is so low under any permitted operating conditions. All copper losses are I*I*R (irrespective of motor type), and so increase with load current and when these = iron loss (and windage) the point of maximum efficiecy is reached. The rotor resistance and inductance can be "referred" back to the stator in the equivalent circuit and can be established by stall and off load tests, in a similar way to short circuit tests etc on transformers.

there are no copper losses on a PM motor if electronically commutated since current is prop. to torque.

As long as current must flow through a (non-superconducting) wire there will be I sq R losses. Even at no load, current must flow if motor is to rotate. Still needs to produce torque req'd to overcome windage, bearing, etc. losses.

Your [q]maximum efficiency occurs when the "load dependant loss = Non load dependant loss".[/q] Not sure about that, never heard of it before, but that doesn't make it wrong. Sounds a bit iffy regarding motor theory though.

I should have said negligable. Yes there will be a small loss, but I assumed no load, no torque so no current, but friction and windage yes so a small loss, its one of my famous approximations.
Its a law, of similar standing the maximum power transfer theory (maximum power tranfer occurs when the source impedance = load impedance) under this condition efficency is 50%. It comes from differentiation and finding the turning points (max and min) of the equation that describes efficiency in terms of iron and copper loss. Its a long time ago since I was forced to derive it, but if I can find a reference to it I will. (if your interested) It may be an approximation for a motor becuase losses are more complex, but for a transformer it holds true.

Lengould, I don't know if this link will work, but illustrates the theory and how to derive it. Its one of those laws I suspect will apply to any energy transfer system, though as an electrical engineer, I have only seen the proof for electrical machines. This document is for dc motors, but the maths does not care what the motor type is.

If you are going to comment on motor theory in the future, please don't question those who understand it unless you do yourself.

As the saying goes, those who think they know everything are anoying to those that do.

http://nptel.iitm.ac.in/courses/Webcourse-contents/IIT%20Kharagpur/Basic%20Electrical%20Technology/pdf/L-40(TB)(ET)%20((EE)NPTEL).pdf

Edit:

It does not work as a link, but cut and paste it into google and it does. Perhaps one of the oil drum gurus can tell me how to create a working link from it.

I would just recommend that you review theory of efficiency calculations. A motor "at no load" by definition is doing zero effective work, but still drawing some power, therefore has zero (eg. lowest possible) efficiency.

That's why peak efficiecy occurs at some load much greater than zero, because at zero load, unless you have designed your motor very badly, the fixed loss (iron) is much higher than the variable loss (copper), hence the efficiency is not maximum. Check it out for your self, i gave you a link.

Its a law, of similar standing the maximum power transfer theory (maximum power tranfer occurs when the source impedance = load impedance) under this condition efficency is 50%.

Sorry, I don't think I'll be listening any further to anyone who applies this sort of logic to electric motors. 50% is maximum efficiency? Ha.

For example, this quote from EnergyStar.gov presentation, http://www.energystar.gov/ia/business/networking/presentations/feb_05_mo...

“Right-size” the Motor
• Choose the correct rating for the application
– Oversized motors have lower efficiency andpower factor
Highest efficiency 75 -100% of rated load

I can only conclude you very little about electrical engineering. The maximum power transfer theory is just that. maximum efficiency does not necessarily, and usually does not, occur at maximum power transfer, that is obvious. If you drew maximum power from the grid, 50% of the energy would be lost in the grid and it would probably burn out.

I was simply saying the maximum power transfer theory is well known, and so is the condition for maximum efficiency, but they are two different conditions. You really need some additional training on these very basic and fundamental ideas. Any good electrical engineering text book will help you.

In that case these motors have been designed to have variable and fixed loss equal at 75%. Dead simple, the law applies.
Over sized motors do have low efficiency, because the iron loss is greater than the variable loss at below optimum load. I can't understand why you can't grasp it.
Whether you like it or not the theory holds true.

maximum efficiency does not necessarily, and usually does not, occur at maximum power transfer

Perhaps your problem is one of definition. Define "maximum power transfer" in your terms. Is that the amount of power one can put into a locked-rotor condition motor for the few seconds before it burns out, or the maximum manufacturer rated power input? If the second, then you're clearly wrong because every properly designed industrial motor is designed to achieve its maximum efficiency at or very near its maximum rated power. Check any other manufacturer's publications if you don't like the Baldor / Reliance website which I referenced.

I think you may be badly confused, mixing motor theory with high-frequency RF circuit theory, where the impedance to power transfer is almost entirely due to the very high frequency effects (often stray capacitances and inductances developed in the conductors and feeder circuit elements themselves and not the load circuit). In those cases, yes, I'll agree that such issues as that which you describe can limit efficiency to a percentage much lower than that of a simple effectively unlimited power source feeding into an industrial motor at very low frequency and with only rated load or less on the motor.

If you drew maximum power from the grid, 50% of the energy would be lost in the grid and it would probably burn out.

?? Perhaps further definition of terms would make this statement appear less obviously wrong? I don't care what power level you're going to draw from the grid you're never going to see 50% of it dissipated in the grid itself. Though that must be some scary sort of test situation. You haven't lived until you've observed a dead short on a 2 MW bussbar in a switch room (as I have), and the results, effectively a lot of vapourized copper and steel. The formulae for calculating exactly the short-circuit capability of a service feeder are well-known. The following quote from Federal Pacific's Transformer Basics details it http://www.federalpacific.com/university/transbasics/chapter5.html

The maximum short circuit current that can be obtained from the output of the transformer is limited by the impedance of the transformer and is determined by the multiplying the reciprocal of the impedance timed the full load current . Thus, if a transformer has 5% impedance, the reciprocal of .05 is 20 and maximum short circuit current is 20 times the full load current.

So the maximum draw one can get from the grid is entirely dependent on the size and impedance of the nearest upstream transformer to the point of the short, and it can be impressive even though eg. a short on a building feeder behind a 2 MW transformer which can drive molten copper right through steel doors (which I've seen) is still only drawing 40 MW from a grid capable of delivering 25,000+ MW in normal operation here. Look up BIL ratings reference switchgear. So what losses the grid would experience delivering its total 25,000+ MW to a single point depends entirely on the design of the nearest upstream equipment, but if that's a code rated transformer, it will suffer no more than 5% losses, not 50%.

But enough of this silly correcting of your errors. Its way off the websites topics of interest.

http://en.wikipedia.org/wiki/Maximum_power_theorem

Thats why I said is you apply it to power circuits, you would burn them out (blow your self up if you please), but it still applies regardless.

A transformer of 5% impedance would supply maximum power into a similar load impedance. After that point more power would be dissipated in the transformer (you do not operate power circuits under these conditions, not for long in any case). If the load impedance is infinite, no power transfer, if the load impedance is zero, no power transfer. Dead simple, again your theory shows serious lacking.

You keep making a mistake common to people without education in electromagnetics, which is confusing impedance with resistance.  Resistance dissipates power, but inductive and capacitive impedances do not.  Most components of the electric power grid (transformers and transmission lines) have a net inductive impedance and will limit current to a maximum without dissipating anything close to the power that a naïve calculation would imply.  There are even more non-intuitive consequences of this which must be taken into account by power engineers, but I won't go into those.

Maximum power transfer theorem applies to complex (R+jX) circuits. It is used to derive maximum torque for a squirrel cage induction motor for example, which is only a "special" power transformer after all. The mechanical output is represented by a variable resistance and the loss by a fixed resistance. Maximum torque occurs when maximum power is dissipated into the total equivalent resistance. As slip increases beyond a certain value, rising XL of the rotor limits this power and the turning point (pull out torque) is reached and this coincides with maximum power transfer into the resistance of secondary cicuit of the transformer (motor). By adding additional resistance (slipring motor) the maximum torque condition can be recovered, though at a greater slip than with a lower rotor resistance. The peak torque is always the same, it just occurs at an ever greater value of slip. This is all basic well documented stuff, I can't understand the problem.

So no confusion between impedance and resistance on my behalf. I also understand electromagnetics reasonably well.

You're obfuscating the issue (how trollish of you).

You said above, and I copy-and-paste:

Thats why I said is you apply it to power circuits, you would burn them out (blow your self up if you please)

No, they would not even necessarily burn out; they could easily self-limit to currents less than what would cause physical or thermal failure.  Of course, a properly protected system will shut down before it fails physically.

Not in the slightest,

There is no way a power transformer with 5% impedance operating at rated voltage would limit the current to survivable levels. It would be an exciting experiment though if you have one at hand.

O RLY?  Do you think the transformer would fail from physical overstress, or would overheat faster than the fuses upstream would blow?  Do you think that the network it's connected to has zero source impedance?

Things like shorted outputs happen, and the hardware usually survives (that's the design criterion of the protection systems).  That's an existence proof.

Fault conditions are an "instantaneous" event and its obvious (even to you I guess) that the protection employed passes a value of I*I*t much less than that required to damage a power transformer, otherwise it would not be protection would it? your just being a clown. Your argument has nowt to do with the fact you cannot operate a power transformer into a load under maximum power transfer (into the load) conditions for long without burning it out. The same goes for operating an induction motor at maximum torque, you will burn it out, but not straight away!!!!!! It has thermal capacity that can absorb some overload and hopefully more than the I*I*t that the overload will allow through, though not always because some folk turn up the overload too high. Its so simple it hurts that you can't grasp it, God help us.

From what I recall from my last installing a 2MVA transformer, the 11kV side has negligable impedance when referred to the 415 secondary. The 40,000 kA or so of fault current is limited almost totally by the transformer leakage inductance and if not protected, there would soon be a mess. The 11kV impedance would not protect very much at all. There is a GEC film showing all this, it's great fun and a big bang at the end.

So you admit that maximum power transfer isn't a design load condition?  That it's neither necessary nor desirable to operate anywhere near it?  And you've been raising this as an issue, why?

You have descended fully into trolldom.  Get lost.

I never said it was. This began when I said to lengould that maximum efficiency occurred when fixed loss equalled variable loss and this idea was of similar standing in electrical engineering to the maximum power transfer theory. Both are well established laws. He took this as me claiming maximum efficiency is 50%. Well, this mistake was made over a 100 years ago in Edison's era. These two are separate conditions and for some reason you stepped in halfway through and totally misinterpreted what I was saying. so I will clarify,

Maximum efficiency occurs when fixed loss (iron windage)=variable loss (Copper). This applies (as far as I am aware) to all electrical machines. It puts limits on operating efficiency, since you can't simply assume using a larger machine at low load (hence reducing resistive I*I*R losses) will improve efficiency. I strongly suspect there is a mechanical analogy for hydraulic motors, ICE etc etc.

The maximum power transfer theory states that maximum power can be transferred from an emf source when the load resistance = the EMF source resistance. At this point efficiency is 50%, not desirable for a power system, obviously because it will you cook the source (Battery) due to loss, but it may be desirable if you want maximum power transfer for a signal. It was widely applied during the days of valve amplifiers, but not transitor amplifiers, because their source impedance is so low they would be destroyed if it was applied. It also applies to power systems, but is not an operating conditon (I never ever said it was) for obvious reasons. It is more complex because the loss in the source may not equal the load because the source impedance is complex (R+jX). However, reducing the load resistance will eventually cause the condition of maximum power because Z will cause the voltage to drop faster than the current rises. This is the point of maximum power transfer and at this point (probably well before) most transformers will be operating in an overload condition.

Its late at night here in the uk, I have tried to put this into words and reason with you as best I can. If you can't agree now I just assume you just want to pick fault regardless of logic. You are getting offensive now, which helps no one. I am disappointed in you telling a fellow engineer to get lost. Calling me a troll, well I see that as a joke, telling me to get lost, can't you do better than that?

I note that you've still not responded to the electric motor manufacturer's efficiency rating vs power curves I referenced at the beginning of this discussion, which clearly put paid to your ridiculous "Maximum efficiency occurs when fixed loss (iron windage)=variable loss (Copper). This applies (as far as I am aware) to all electrical machines."

This whole discussion has been about your refusal to accept taht your hypothesis does not apply to electric motors or power grids, as I keep proving repeatedly.

C'mon back if you ever get out of first year.

I did look at your information and its basic "sales brochure" performance curves with a few elementry calclations on cost saving, nothing I did not know already prior to reading.

I'm at a loss by how you draw your conclusions from the graphs, which don't give any information about specific loss. For instance you cannot conclude at peak efficiency for each curve what either iron or copper losses are. I don't know what your on, but I suggest you go and get treatment for it. I can only repeat, buy yourself a good book and will will find out I was right all along and your tantrums can cease.

EDIT,

Here is the mathematical proof that max eff. occurs when fixed=variable.

c=constant loss
b=coefficient for I loss
a=coefficent for I*I loss
n=efficiency

so loss=aI2+bI+c

n=(powerin-loss)/powerin (by a bit of manipulation can rearrange to)
n=powerout/(powerout + Loss)
powerout=VI cos(phi)

By substitution you get

n=V*I*cos(phi)/((V*I*cos(phi))+aI2+bI+c)

to find max (or min), differentiate with respect to I and make equal to zero. This is a quotient because both numerator and denominator are functions of "I", so one has to use the quotient rule VIZ

dn/di= v*du/di - u*dv/di / v2

where v=((V*I*cos(phi))+aI2+bI+c)
and u=V*I*cos(phi)

You end up with quite a few terms on the numerator divided by the orginal denominator (v) squared. But the terms cancel down to this

dn/di=((c-aI2)/(denominator)2)* v cos(phi)

From this, it is obvious the equation is zero when c=aI2. Further differentiation can prove whether this is a maximum (turning point) or minimum (turning point), but it is a maximum. It would be quite easy to demonstrate this by a small program written in Q BASIC of MS Excel etc.
It is curious that the "bI" term cancels (as it has no meaning) leaving only constant (iron) and I2 (copper) terms, which is why it is dangerous to rely on intuition, rather than sound mathematics to demonstrate an idea is true. It also demostrates conflicting requirements when designing motors and transformers and gives one a good understanding as to why claimed efficiencies are often not relised in the real world.

Lengould,

I asuume you have accepted the max efficiency proof, since you have not disproved it, here is empirical proof of the max power transfer theory. Its written in BASIC, so you will need a copy, easily available from older Windows and late DOS packages or from the web. You will see that max power to the load is when the impedance of the transformer is equal to the load resistance, though not a permitted operating condition in power supply systems it is an important and well known concept. As mentioned above it is used to prove the maximum torque condition in the induction motor

Mathematical proof is derived in a similar way to my last example using differentiation.

CLS
SCREEN 8
t = 0
R = 0

WINDOW (-0, -30)-(1000, 50)
LINE (0, 0)-(1000, 0)
R1 = 100 'transformer winding resistance (referred to secondary)
X = 100 'transformer leakage reactance (ditto)
v = 100 'secondary voltage

FOR R2 = .1 TO 1000 STEP .1 'secondary load resistance
ztotal = (X ^ 2 + (R1 + R2) ^ 2) ^ .5 'secondary impedance including load
z = (X ^ 2 + R1 ^ 2) ^ .5 'secondary impedance excluding load
I = v / ztotal
power = I ^ 2 * R2
dp = power - powerprev 'dp is incremental change in power
powerprev = power
IF (dp / .1) < .00001 AND dp > -.00001 THEN PRINT "load resistance="; R2: PRINT "z="; z
PSET (R2, power), 10
'PRINT dp
PSET (R2, dp * 10)
NEXT R2

What I objected to was your claim that electric drivetrains wouldn't scale because of limitations on rare earths for magnets.  True, induction motors have greater losses due to slip; however, they are rugged, cheap and need only iron and conductors.  There will be some price point where the savings on RE magnets will buy enough batteries that it yields the same utility.

Fair comment, though I don't recall saying rare earth magnets were rare and would prevent scaling, because many rare earth elements are not infact rare. All I suggested was we would end up with these motors that require position feedback, just as ICE's have been blighted by unecessary complexity to gain ever dwindling emissions improvements.

I am a big fan of the good old Sqirrel Cage induction motor due to its simplicity, but again folk are forever being encouraged to operate them closed loop, which is a sales driven gimmick in many cases, but its happening in the real world. Closed loop is sold for its low speed stability and zero speed maximum torque capability, none of which are required for a train locomotive, but may be saleable feature for a road car, to hold on a hill for example. Complexity is often market driven, engineers simply respond to the market department's demands.

Audi are advertising a 700 mile range for one of their models at the moment, You state this is not required in an earlier post, I would not disagree with you, but its what people will buy that counts.

Does this put us nearer any sort of agreement?

The 700-mile range figure appears to pertain to the VW Blue Sport, and from what I can tell it's about the fuel economy (50 MPG).

I'm not opposed to a car with 700-mile range; I've done well over 700 miles on a tank myself.  But if your concern is operating cost, swapping batteries every 100 miles to get 5¢/mile energy cost would do just as well.  Plugging in at home so you never need to visit any kind of service station for your typical week of local driving would be a winner too.

I don't think I agree or disagree with you; I think you were just missing the point. ;-)

Let me disagree on a few points.

"I doubt a battery charger is more than 90% efficient."

Cheap charges aren't efficient, but when you start talking about several kWh spent every day, an expensive charger starts making sense. One could make a more than 90% efficient charger, it would just be expensive.

"Industrial drives are about 95% at rated output, but this falls to zero as output frequency tends to zero."

That is a quite good reason to use some reduction gear between the motor and the wheels. Ok, a normal car motor operates on a much lower frequency than a normal electrical motor, but there is a very big body of knowledge on how to reduce the electrical motor's rotation into something useful.

"There is no infrastructure to support electric cars, despite what those in fantasy land claim."

Now, I agree with that. But I also think that no sane person will construct the needed infrastructure before there is demand. That is one of the reasons plug-in hybrids look so good.

"It has been regularly quoted that compressing hydrogen to 200 bar that is consumes one third of the energy contained, so you are being generous here."

I agree, and also think that he is quite generous with the ICE. To make it short, the graph seems generous with all the analyzed alternatives...

Marco, I'm not sure we disagree by that much, a few percent perhaps. I just want to make the point all loss must go into the pot, and cannot be ignored whether efficiency is 90% or 95%.

The problem with making battery chargers evermore efficient is the law that maximum efficiency occurs when fixed losses = variable losses and this is the bugbear of electrical machine design. If you make a transformer too big iron loss dominates, too small copper losses dominate. I suppose constant current charging may be the answer so the load is more predictable and the design can be optimised. But only quite large transformers exceed 95% due to another law; VA is proportional to fourth power of the dimensions, whist losses the third. VA throughput therefore increases faster than loss and as size increases efficiency tends towards 100%. It may become essential to make the chargers draw sinusoidal currents from the supply using an active front end, this will be another headache and source of loss. This would become an issue when large numbers are required, to avoid harmonic distortion in the supply voltage.

I agree gearing is a way to mitigate the above problem within the motor and is "impedance matching" the motor to the load. The only issue with gears is they are too a source of loss, roughly 5% per mesh. This is the whole issue with electric cars, they are such a varible and unpredictable load that there is no optimum condition for which to aim. BUT, "JOULE" in a post below is suggesting we don't use gears!

On a second and separate issue, rail transport is ideal for electrification and the load is far more predictable, by virtue of the way trains operate . Does anybody know why the uk is planning for diesel locomotives in its recent investment anouncement? I picked up on the point that the engines would be sourced from the UK or Europe, so have assumed diesel is planned.

AC Propulsion claims 93% efficiency for the AC150 Reductive charger.  This is quite adequate.  It requires no additional inductors because it uses the motor windings; I assume that it can also run at near-unity power factor.  This would appear to address all your objections.

93% will do nicely, as long as its correct and included in the pot. But as I suspected its a maximum value, the minimum quoted is 50%.

I hate to poop on your party Partypooper, but based on current offerings, which are few and far between, charger efficiency varies from about 50% at low power/voltage to 95+% at higher voltage/power ratings, so it can be pretty crappy but is would probably be at 90+% for most apps. At the kind of voltage/power output needed for air conditioners (barring the kind you put in a window) and electric dryers, the efficiency is somewhere around 95%. The efficiency of an electric motor is also relatively low at low speed/high torque like you mentioned, at ~75% (same source as above) and goes up to 91% at high speed/lowish torque, w/ an average of around eighty something percent.

Course, in the spirit of pooping on parties, a conventional vehicle will only see 40% efficiency very rarely, and will in fact spend most of the time around ~10-20% efficiency, so there are definitely greater losses there as well. The Prius for instance, has a peak efficiency of 230g/kWh. With a gallon of gas at ~6.25lbs/2834g, this means the Prius can at best get 12.3kWh of mechanical energy out of a gallon of gas. A gallon of gas has about 36.6kWh of energy, so that means that a the engine in the Prius, one of the most efficient cars out there, operates at ~34% efficiency. Course, since gasoline engines are so lossy at low load, going from ~230g/kWh or 34% efficiency, to 400-600g/kWh, which is ~13-20%, we can see that if even one of the most efficient cars seen today, which actually cycles the engine on and off and saves that energy in a battery pack for later use because it's more efficient todo that than run at low load, can barely hit 34%, then the average car is probably somewhere around 10-20%. City driving is pretty close to a 10-15% average for most vehicles, and highway is probably around 15-20+%, with an average of around 15-20% give or take.

Since the only way to significantly improve fuel efficiency *given consumer attitudes is via hybridization, and even then we're only going to see something less than ~35%, since the emissions systems still has to be lit off and the engine warmed up, from the POV of efficiency there's no point not to stick a larger battery pack in and have a PHEV or pure electric.

*Arguably, if we could convince everyone to drive cars with 6-8 speed manual transmissions (maybe CVTs ala the 3L Lupo) ,1-2L engines w/ idle shut-off, no A/C, and so on, regardless of vehicle size, then we could see ~30+% average efficiency but I think it's more likely people would accept limited range from electrics before they accepted all that.

Rolf, Waffle you do

You first paragraph mearly repeats what I have said or already aknowledged above. I've learnt nothing new from you here.

No where do I mention, use or imply 40% efficiency for an ice or anything else.

230g/kWh is no measure of efficiency. Efficiency is (energy out/energy in) not grammes/kWh, that makes no sense.

Vehicle efficiency is always zero. All the energy put in is lost as heat and no net work is done so energy out is zero, ie 0/energy in = 0. Engines motors and transformers have an efficiency, cars don't any more than plastic moulding machines do.

The ice has an efficiency range from zero (at idle) to an at very best of 40% (at maximum torque)

The only way you'll see an ICE operate at 40% efficiency is a large steady-state diesel running constant rpm at it's design rpm. An otto cycle in a car will be lucky to get over 20% any time, much less many times. Even a diesel car will have to eat the efficiency hit of stopping / braking / idling / restarting, so can't claim near ideal achieved on dynamo.

Correct, 40% will not be achieved, it is an ideal operating condition maximum possible if your an optimist. The same applies to electric transport systems as well, including motors. The 92% motor efficiency will not be achieved in a variable and unpredictable load application suc as an ev, as many here keep claiming.

Who is claiming that 92% efficiency would be achieved consistently? If you read my charger link you'll notice a load/speed/efficiency map that shows efficiency ranging from the 70s to the low 90s, with an average given most driving cycles of around 80-85%.

Anyway, while pointing out the difference between real world an optimal numbers is great IMO, you shouldn't just focus on electrics since that presents a very biased viewpoint. An EV may be at ~80% of what the OP mentioned on average due to a charging efficiency of ~95%, and motor efficiency at ~85%, or whatever the specifics are given real world data, and using the same methodology a conventional vehicle may only be at ~30-50% of what the OP mentioned on average

You first paragraph mearly repeats what I have said or already aknowledged above.

Not exactly, since you stated that you doubted a battery charger is greater than 90% efficiency, even though companies make chargers that can easily result in 95% efficiency given common household circuits.

I doubt a battery charger is more than 90% efficient.

No where do I mention, use or imply 40% efficiency for an ice or anything else.

That wasn't directed at anything you posted, just the OP's assumption. I figured I would toss it in the same post in the spirit of party pooping. ;)

230g/kWh is no measure of efficiency. Efficiency is (energy out/energy in) not grammes/kWh, that makes no sense.

The ratio of energy output compared to the energy in the gallon of fuel is the efficiency of engine operation/output. If an engine can only make ~12kW with a gallon of petrol, and there are ~37kW of energy per gallon of petrol, then it's efficiency is the ratio of those two. Since it's more or less static how much energy is in a gallon of petrol, then all we need is the fuel consumption and power output of an engine, which can be in g/kWh or whatever other similar units, in order to calculate efficiency. The fuel consumption/power output of an engine tells us how efficiently it's operating.

Vehicle efficiency is always zero. All the energy put in is lost as heat and no net work is done so energy out is zero, ie 0/energy in = 0. Engines motors and transformers have an efficiency, cars don't any more than plastic moulding machines do.

Sure they do. The reason why we would include an automobile (glider if you will) is that given some initial conditions driveline efficiency does depend on vehicle characteristics. Tossing a 2L engine in a larger vehicle would result in greater engine efficiency (up to a point) all things being equal since it would have to be loaded more heavily, as opposed to a 4L engine that would incur much greater pumping losses. The lower variability in efficiency given load, among other reasons (such as idling like you mentioned), is why electrics are so much better than conventional vehicles in terms of energy efficiency.

The ice has an efficiency range from zero (at idle) to an at very best of 40% (at maximum torque)

Peak efficiency tends to to be where pumping and friction losses are minimized, which may or may not be peak torque. It really depends on the specific engine you're referring to. Even then, production engines with a best of 40% BTE are rare in automobiles, w/ the very best being a VW engine at 38% BTE IIRC, and most being at 30-35%. Of course that isn't even the whole picture since we still have to warm the darn thing up and light off the emissions system. In terms of real world performance the more efficient vehicles like the Prius are somewhere around 25% efficient given typical use, and others tend to be south of 20%.

It is expected, all things being equal, that that number will double by 2031.

So, in a world increasingly staring serious CC in the face and, if the economy were to drag itself out of recession (and the collaspsing motor industry actually decided to increase production again) close approaching peak oil, what you are proposing is new ways of burning up more fossil fuels?

By 2031 I imagine the whole of the worlds fleet of cars will have been replaced. This is likely to happen whether you like it or not. In my opinion, its best to replace the existing car fleet with the most energy efficient vehicles that can be designed and built, and that will include cars that last 15 to 20 years and which can be largely recycled, running on renewable energy. Too bad that you don't agree with me.

Well no, I don't agree - I think that structural energy demand reduction is "essential to the short-term survival of our industrial economies." People walking, cycling, catching the bus and taking the train and leaving their cars, petrol, diesel, hydrogen or electric at home! Or if they do still need to travel by car, two people sharing a single current car model will be far more efficient that those two each driving their own electric cars. Taxes on single occupancy could kick start that one, whereas waiting for all these millions of electric cars to sort us out will take .. err, how long?

In other posts my views in favor of imposing speed limits, power and engine size limits, tradeable energy quotas are well documented. Two people sharing an electric car or plug in hybrid would be much better that 2 people sharing a gasoline ICE car - IMO. But this post is not about that, its about energy efficiency, which I think must lie at the heart of everything we do. It seems you favor some version of a mythical paradise where all the houses, shops and places of work are somehow just magiced into one place so that folks can walk or cycle everywhere. Sounds great, you'll become very popular saying things like that, but its wholly impractical. Whilst I agree entirely that our infrastructure design is dreadful, and needs to be redesigned and replaced over the coming century to be much less energy intensive, that just ain't going to happen overnight.

The UK is in a mess in terms of indigenous oil and gas supplies and hence our balance of payments is looking increasingly unhealthy as (I seem to remember) you have so excellently pointed out in some of your other posts. To squander these things by replacing the UKs existing cars sounds well intentioned, but I cannot see it as the answer.

Well unless the economy tanks completely (which it may well do and this may deliver your mythical paradise) then the global car fleet will be replaced whether you like it or not. Might as well replace it with the best energy efficient technology, running on viable renewable energy IMO.

Top Gear makes me scream

Why do you watch it then?

"trespassers in the motorcars domain, they do not pay road tax and therefore have no right to be on the road, some of them even believe they are going fast enough to not be an obstruction. Run them down to prove them wrong."

The serious point here is that in the UK it is apparently illegal to cycle on the pavement / sidewalk. Where I live, we have mile upon mile of broad pavements used by occasional pedestrians. Meanwhile, occasional cyclists are forced to take their chance on over-crowded roads. Its insane. In Aberdeen, and throughout Scotland they have painted hundreds of miles of lines down the side of roads and planted bicycle signs on them and are making believe that they are creating cycle lanes. Often 50 cm wide, with cars parked on them, and paint worn off after a couple of years, this is a total waste of money - all done in the name of Greenwashing Council policy.

By 2031 I imagine the whole of the worlds fleet of cars will have been replaced. This is likely to happen whether you like it or not.

lATER ...

Well unless the economy tanks completely (which it may well do and this may deliver your mythical paradise) then the global car fleet will be replaced whether you like it or not.

Are you really sure????

I have questions:

Who will make all these cars? General Motors? British Leyland? Fiat?

Where will the 'car owners' drive these millions of automobiles to and from? Suburbia? Back and forth to work in offices? Will they drive to the mall? Will they drive out to the country- side to scavenge for something to eat?

How will these millions of cars be made? In factories or by blacksmiths? What is the EROEI for an auto factory? Do you know how much it costs to build an auto factory? How much it costs to build - and fix - a medium sized road? Where is that money going to come from?

Where will these cars be made? Will they be made in Japan? How will they arrive in the US or Europe, how will American cars arrive in other countries? On sailing ships? What is the EROEI for a clipper ship? What about the wind turbines and grid and backup power for the humans? Do we get some precious power or is it set aside for the autos?

Most people think about the economy as a discrete thing, like a grand piano or a cuckoo clock. The economy is too complex for such analogies. You can only look at the economy a little at a time; a point here, a point there. Add the points together and you have something like a cloud. A cloud is comples and interconnected. However, if one part of a cloud is disturbed, the rest of the cloud continues as before. A cloud is unresponsive. The economy is very sensitive.

A better analogy is an organism, a bear or a chipmunk. Nobody can see an entire chipmunk or all the parts of it, only a bit here and another bit there. An organism is responsive. If a bee stings the bear on the foot, the entire bear reacts. Tiny bee, giant bear. Cut the foot off the bear, and something must be done or the bear will die. Now, the bear has gotten incredibly large, from devouring everyting within reach. At this point, it is eating itself! Not necessarily by swallowing its tail, but by digesting itself from within. For years and years it has been doing this aelf- digesting and for a long time it has been; '... so far, so good!'

Now, the digestion process is advanced. People say, "The eocnomy is collapsing!", or, "the economy is broken!", or it is "tanking". It's considered sick or ill or needing a jump- start or some other 'treatment'. Rather, the massive, all engulfing economic bear is manifesting advanced self- digestion. There is nothing really 'wrong' with the bear, in fact it is exactly as it should as it desperately tries to save itself.

The economic bear must either stop the digesting or it will die. It will completely devour itself.

This is what 'sustainability' is all about. Some action proceeds - like beating a family member with a baseball bat - until it cannot anymore; the action morphs into something else that may be sustainable or it stops, completely.

Period. There are no exceptions. There is no such thing as 'more sustainable' or 'less sustainable'; there is sustainable and there is everything else. Being 'more sustainable' is like being 'more pregnant'. Sustainability can be considered a natural law, one of the few metaphyxical principals that reaches into the world of the 'REAL'!

I call thes 'THE IRON LAW OF SUSTAINABILITY'. It is the mainspring that drives markets; PRICING by supply and demand. Markets attach values to actions and when these become manifestly unsustainable, the market reacts. The market has declared that the auto industry is unsustainable and this declaration has been made bu the market pricing the industry into the toilet.

Like the bear digesting itself ... the car business has succeeded to the point of failure. In light of this 'market pricing', the issue of what kind of motive power is 'under the hoods' of the different cars is irrelevant.

The only way the auto industry can possibly survive at this stage of the digestion cycle is to stop making cars. It can make something else - streetcars) or military vehicles (In very limited numbers) or sailing ships. Otherwise, it will make fewer and fewer more and more costly (to itself) autos... and will eventually disappear.

This sounds ironic, but we live in ironic times and have a wickedly ironic economy. GRRRRLL!

steve from virginia -

You apparently are making the same error a lot of people around here seem to make: using the concepts of EROEI and energy efficiency interchangeably. They are not the same thing.

If used rigorously, the term EROEI should only pertain to the production of primary energy sources, e.g., fossil fuels, wind power, solar power, etc. Though some may disagree, I maintain that EROEI does NOT apply to downstream processes or activities that consume energy. That's what the term 'energy efficiency' is for.

For example, one can legitimately talk about the energy conversion efficiency of a coal-fired power plant or the energy efficiency of an automobile, but applying EROEI to either makes no sense, because both are by their very nature consumers of energy, i.e., they are not intended to 'return' energy but rather to use it. In the case of the power plant the energy content of the coal is used to produce a desirable product: electricity. In the case of an automobile the energy content of the gasoline is used to produce a desired activity: transportation. In either case one can be more efficient or less efficient in energy usage, but one cannot produce an energy 'return'.

This may be nit picking, but I have a pet peeve at the way EROEI is being increasingly used in ways it has no place being used.

We should be careful not to bring this metric too far, but it is very useful to multiply the upstream EROEI with downstream efficiencies up to the point of useful work.

It shows us things like:

- if we double the end use efficiency, the effect of halving EROEI (even in 0-1 ratio/percentage terms) isn't that bad.

- crap EROEI to start with will be of very limited use even with very high downstream efficiency (traditional corn ethanol in a moderate climate).

- having very high EROEI upstream but crap use efficiency becomes a more serious problem when EROEI is declining, and this is not evident now because today's oil high EROEI masks the dismal end use efficiency.

There are other aspects that must be taken into account such as exergy and externalities, but as far as single indicators go, I think upstream EROEI x downstream efficiency is a very useful metric that need not be incommensurable.

Well unless the economy tanks completely (which it may well do and this may deliver your mythical paradise) then the global car fleet will be replaced whether you like it or not.

Merely replacing the global car fleet, but not planning for it to grow represents massive global recession over the next 2 decades.

What is the EROEI for an auto factory?

ERoEI is a measure of efficiency of primary energy production.

Who will make all these cars? General Motors? British Leyland? Fiat?

Mercedes Benz, Volkswagon, Audi, Honda, Toyota?

There is no such thing as 'more sustainable' or 'less sustainable'; there is sustainable and there is everything else.

The world and the human race is not going to convert to hippie style commune living overnight. Adaptations will occur that move us from unsustainable towards more sustainable. To gain the goal of totally sustainable will require a massive reduction in population from current levels. Until that happens we will most likely hobble on.

Let's look at it another way.

This post is not intended to provide a rigorous analysis of automobile efficiency but aims to provide an overview of the efficiencies of different drive system - fuel combinations.

The procedure followed is to identify the main energy efficiency factors and to multiply these to provide the overall efficiency in %. The efficiency of fuel source is based on ERoEI where:

ERoEI = energy procured / energy used to procure energy
Efficiency = (ERoEI-1)/ERoEI

I have not considered the energy embedded in the vehicles and energy losses downstream from the motor in the drive systems. The objective is to highlight the major differences between the 4 fuel / drive system combinations.

You are using EROEI a certain way; I am using EROEI same way. You are not considering embedded costs - auto plants, auto shipping, infrastructure 'development', auto- plant workers' energy liabilities, auto customers' total lifestyle- related energy costs, etc. You have to! These are the largest part of the industry's total energy budget! The terms used to define the embedded costs are the same terms used to describe operational costs.

How can embedded costs not be considered? They are real, these costs are weighing on the existance of the industry, not operating energy costs per vehicle. The effective difference between 20 miles per gallon (or equivalent) and 30 miles per gallon (or equivalent) is trivial, compared to the energy footprint of the entire industry. You say that all things are equal and that relative effiencies are important; I say that the industry is deader than a vampire with a stake though its heart, who cares what that industry's products' operating efficiencies are? The operating efficiency of the industry as a whole is fatal to it!

Once upon a time, operational costs per vehicle were all that mattered, the embedded costs were externalities; rendered 'off the balance sheet'. Not any more; the costs are now on the books and are being priced in the marketplace. this is why the auto makers are going down the drain. This is being done by the customers of the auto manufacturers. Not by me.

Neither Toyota, Mercedes, Honda or Audi will be able to dodge the market effects of the energy balance sheet, which prices the energy costs into currency.

Whether the result of this process is a 'Greater Depression' and hippie lifestyle commune living or some other outcome is irrelevant speculation. The implied fear- mongering is intellectually dishonest. The energy return calculations are being made in the hard- nosed, ultra right wing, Pro- Reagan/Thatcher 'Austrian school of Economix' marketplace, by dim- witted, TV embalmed, advertising brainwashed, Jesus soaked inhabitants of the American 'Moron Crescent'. This is happening right now, whether you like it or not!

(James H. Kunstler, take a bow!)

The only accurate way to consider the industry's energy use is to do so in its entirety. Doing so leaves that industry bankrupt. The industry does not return any real work on its vast energy inputs; autos aren't necessary for human life, they are conveniences or toys, status symbols, signifiers of 'wealthy-ness', rolling fornicatoriums. They are also crash coffins, space destroyers, pedestrian/cyclist killers and maimers, air polluters, water polluters and sprawl enablers. The only work pruduced by the industry is from fire engines, ambulances, some delivery trucks, military and police vehicles, and a few other minor types. The industry is a colossal energy sink.

It cannot survive in its current form making fire engines - or electric cars. It is capitalized to produce millions of wealth signifiers at scale. You can get any color as long as it's black ...

The argument that the industry is necessary for employment or economic function (growth) - for the benefit of those 'little people' of course - belongs in the Wall Street Journal or Larry Summers' office. At bottom, the industry exists for its own sake. Its participants - including its consumers - toil to benefit the industry and its masters, rather than any greater purpose.

Finally, from an economic and practical standpoint, discrete ER separated from all of its downstream processes (EI) is impossible. This is because ER is not static, it does not - and probably cannot for long - exist out of use context. Energy does not visit another dimension after it is 'returned' but is immediately (or rapidly) forced through all of its utility functions until it becomes waste; heat or gasses in the atmosphere, or chemicals in the ocean. At every step in the cycle there is an energy cost that must be paid which requires its own energy investment. Instead of EROEI, it should be Net Energy Return on the Total Energy Budget.

Energy cannot be separated from its use (or utility) context. Here, utility means the subjective value of the work derived from the energy; this is very close to the 'classic economic definition' of utility. Some utilities are more energy- useful than others. Energy used to create solar panels is has a greater return - and a greater utility - than using the same energy to drive in large loops from mall to mall to mall and then to a bar.

steve from virginia -

I think it is a fundamental flaw to attempt to account for everything when doing an EROEI analysis for a primary energy source, an analysis which only has much meaning when attempting to compare one energy production scheme with another.

When one starts going back more than a few levels, i.e., after the actual direct energy inputs have been accounted for and after the energy content of materials of construction plus construction-related energy have also been accounted for, then the analysis gets less and less fruitful, as one very rapidly gets into an energy allocation game. And once one starts doing that, then all sorts of arbitrary assumptions and value judgements come into play, and thing become more and more murky rather than more clear.

Regarding your example of the auto plant, the purpose of an auto plant is not to 'return' energy, but rather to product automobiles. It can do this efficiently or inefficiently, but what it cannot do is to produce any net energy. Trying to account for every BTU of energy that has been somehow associated with that auto plant can get unwieldy real quick.

For example, many years ago when I was in the environmental consulting field, I was involved in a project at a large auto plant in Michigan. Over the course of the project I probably made a dozen round trips from Boston to Detroit. Would you have us include a portion of the fuel used on the plane that took me to and from Detroit? Or the gas I used in my rental car once I got to the airport? That fuel was expended as a direct result of the operation of the auto plant. You see how fast it can get downright silly.

There is nothing wrong with cutting off the boundaries of analysis at some reasonable level of detail when calculating EROEI as long as one is consistent when comparing several different alternatives. One has to not lose sight of what was the purpose of the EROEI analysis in the first place. It is NOT to chart every single energy pathways of everything that takes place in the entire US economy.

Joule, I understand what you are saying, but I don't see how an energy balance sheet can be useful if it leaves out closely related energy costs, which have to be paid in energy ...

I presume you and I agree that any energy cycle is finite and closed where the energy invested + work done can never exceed energy available. (Is always less on account of thermodynamics and inefficiencies.) I agree there is nothing wrong with setting different boundaries of analysis @ some level of detail when calculating EROEI; I don't want to not calculate in energy what financial marketplaces calculate in money.

In other words, your flying from Boston to Detroit is an energy cost that has to be measured against some energy return, somewhere. Why not the auto plant? Since they are the beneficiary, it makes sense to assign the energy cost there, rather than arbitrarily placing that cost elsewhere.

Your money cost of flying from Boston to Detroit was assigned to (and hopefully paid for by) the auto factory. Why not the energy cost?

The only way I can see to practically measure Energy Return on itself is to measure worldwide TOTAL ER for all the different forms of energy extracted. At that point, all the subsequent energy expenditures - and all cross connections and interfaces would be downstream from original extraction. How would anyone be able to measure Energy Return worldwide, minute by minute? You would have to meter consumption, since 'energy' doesn't manifest itself unless it does actual work. You would be measuring Energy Invested (costs) and extrapolating from that point what the Energy Return actually is ... by calculating (assuming) that Energy Return is somewhat greater than EI. In other words, the EROEI model is useful at the extraction level but very analogous since you can't measure energy, only work. It becomes less analogous at downstream levels but provides less useful information, since its output and a simple extrapolation are very similar; it doesn't appear to scale very well.

Very interesting metaphysical process, I like it.

And ... okay, I'll admit it, the auto industry appears to me to exists sole-ly to consume (waste) fuel. The process is what matters, the product - means to that end - is irrelevant. If the industry could simply burn the fuel in furnaces within the factories and make some money by doing so, it would!

steve from virginia -

Not to belabor the point beyond which it already has, the auto plant does not 'exist solely to consume (waste) fuel', but rather it exists for its obviously stated purpose: to produce automobiles. It consumes fuel (and money) in the process of making automobiles.

Yes, if an auto company 'could simply burn the fuel in its furnaces within the factories and make some money by doing so, it would!'. Well, so would I if I could, but I can't, and neither can they. Nobody can. The consumption of fuel in the production of automobiles is but one of many inputs required to produce automobiles. You can't make something without expending energy in one form or another.

I think you have it backwards: the final product is what matters, and the process that caused it to be created is what makes it more worthwhile or less worthwhile, depending upon how wasteful that process is, not only in terms of energy but also in terms of various materials and other inputs. The question of whether automobiles should or should not continue to be produced is largely irrelevant to this discussion, which I though had to do with the right and wrong way of using the concept of EROEI.

Joule,

I think you have it backwards:

Of course I have it backwards. I always have it backwards.

Through most of the (now ending) age of cheap oil, the main problem for producers was over-supply. That is why it was the age of cheap oil.

One solution to the problem of overproduction of oil was the car industry. Cars could insure an ever increasing demand for the less desirable (for industry) grades, leaving the more desirable diesel for industry and (rapidly industrializing) farms.

The primary economic purpose of the auto industry, imo, has always been to soak up excess production of oil and petrol in particular.

This model is, of course, falling apart as we speak (or type).

And, of course, everything is bass-ackward

Through most of the (now ending) age of cheap oil, the main problem for producers was over-supply. That is why it was the age of cheap oil.

Think about the 'Great Streetcar Conspiracy' where a number of auto and petroleum companies bought up streetcar companies across the US and put them out of business:

Between 1936 and 1950, National City Lines bought out more than 100 electric surface-traction systems in 45 cities,[2] including Detroit, New York City, Oakland, Philadelphia, Phoenix, St. Louis, Salt Lake City, Tulsa, Baltimore, Minneapolis, and Los Angeles,[3] and replaced them with GM buses. American City Lines merged with National in 1946.[1]

http://en.wikipedia.org/wiki/General_Motors_streetcar_conspiracy

The effective difference between 20 miles per gallon (or equivalent) and 30 miles per gallon (or equivalent) is trivial, compared to the energy footprint of the entire industry.

I doubt that. MacKay (Sustainable Energy — without the hot air) mentions about 30,000-75,000kWh per vehicle from two sources depending on whether virgin or recycled materials are used. That would mean even a Prius would burn as much energy via gasoline as it took ( plants, raw materials, etc) to build it in the first place at around 100,000 miles on the high end and ~50k miles on the low end. At the American fuel efficiency average (largest oil consumer in the world) average this shrinks to about 34,000/17,000 miles, and so on. If we all have Priuii(?) or similar in the future, given that the embodied energy required comes from different sources, and if we continued to only use conventional vehicles and/or HEVs, from the POV of a declining oil supply vehicles that pulled 100mpg would still be a worthwhile upgrade even if current fuel efficient vehicles do make it to 300k+ miles.

There's also something of a knock-on effect since smaller vehicles that are more fuel efficient also require fewer materials, and in this context the less complex a vehicle is the more likely it is to last longer overall, so EVs would have the added benefit of not needing replacement as often as well as requiring about ten times less "home grown" electrical energy than a conventional vehicle requires in "home grown" energy from oil to travel the same distance.

The ILEA found that a midsize (today's large) car only required about 10% of its total lifecycle energy cost in manufacturing; most of the rest was for fuel during its operation.  A Prius might require more energy to make, but its lifespan is generally greater, so the effect is probably a wash.

It will take a lot of efficiency improvements to hit the point of diminishing returns for vehicles.  This goes double for the carbon balance, where shifts from liquid fuels to electricity (esp. wind-generated) can cut carbon per unit energy.

Merely replacing the global car fleet, but not planning for it to grow represents massive global recession over the next 2 decades.

Adaptations will occur that move us from unsustainable towards more sustainable. To gain the goal of totally sustainable will require a massive reduction in population from current levels. Until that happens we will most likely hobble on.

Exactly what do you mean by 'hobble on'? Yes, the powers that be will try to preserve the holy right of money to make money for as long as possible, meaning that they will try to maintain economic growth for as long as possible. Granting people unearned purchasing power in the present in exchange for a larger amount of purchasing power in the future requires growth in the production and sales of total use value. Without such growth financial investing becomes a mean of robbery which collapses when it becomes obvious that the promised excess productivity is not going to materialize.

You say that the human population of the earth is already massively unsustainable, and yet you suggest that we should 'hobble on' trying to avoid a recession (i.e. trying to keep on growing our total economic output). In what universe does this statement make any sense? If we have been living beyond our means and spending down the earth's capital, then we have no choice but to retrench, to live more simply. Yes, improved efficiency can help to reduce the amount of retrenchment necessary, but to insist that we have to go on pursuing growth because within the current economic/political order a recession will have very unpleasant consequences represents a head in the sand refusal to face reality.

I tend to agree with the kilted green. Euan has shown a distinct aversion to sustainable economics or societies, in the past, even though he understands the limits of nature.

But it is optimistic in the extreme to suppose that the worlds car fleet will be all electric by 2031. That is barely 22 years away and it takes, I think, something like 15 years to replace the fleet, with extra cars being added all the time. So each iteration might make some inroads into electric car utopia, but it would take a lot longer than 22 years for a significant portion of the fleet to be all electric. And that is assuming similar kinds of societies and economies to now. People need to sell their car (usually) in order to buy another, but will that even be possible for many people?

The kind of societies envisioned by having all electric cars for a billion people are probably pie in the sky societies.

This earth is limited. The energy and resources available for us to use without damaging the environment is limited, even if we damage the environment, they are limited. Unless authors understand this, we won't get any useful articles from them, about how we should plan future societies or prepare for a more resource limited future. Quotes such as, "It seems you favor some version of a mythical paradise where all the houses, shops and places of work are somehow just magiced into one place so that folks can walk or cycle everywhere." are not helpful. The implication is that any kind of society that requires reduced consumption just ain't gonna happen. If it doesn't happen, bye by civilisation.

As Steve from Virginia mentioned, "more sustainable" is meaningless. Something is either sustainable or it is not. Current lifestyles, using 1.2 earths and rising, are not sustainable. Even if we all drive electric cars.

I agree with what quite a few people on this thread seem to be saying. It seems to me that efficiency is only a small part of the overall picture of our ability to use electric cars as a replacement for what we have currently. Electric cars, with big batteries, have big upfront energy costs. Maintaining roads have big energy costs. Building new long-distance high voltage electric grid to transport wind energy further and distribute it better also has big energy costs, and I question whether it will really happen on the scale needed, given funding challenges.

In a poorer society, I doubt that we could really manage this transition. Even if we could, it is not sustainable in that we are continuing to use vast amount of fossil fuels for making the cars and maintaining the system. It will necessarily crash, just as our current system looks ready to crash. If Euan is correct in saying that we will be able to make this transition, it seems to me that at best, it will just push the crash out a little further. After the crash, it seems like we will be worse off than if we had never attempted it--more pollution, fewer remaining fossil fuel resources, and no real plan for how we will live without fossil fuels.

Dear Euan.
Interesting points for discussion, but where will we get the electricity from? - Not just for hybrids or all electric vehicles but for basic living? (I speak not just of Scotland, but also the UK as a whole). As you are most certainly aware, the recent cold spell has punished our meagre gas storage reserves. Nuclear power at least in Scotland is still off the agenda and wind is at a very immature stage. Tidal is still on the drawing board and of course Coal is now unacceptable to the pro AGW people. Add to this that we now no longer have any money after the bailouts and less tax revenue and increasing numbers of people becoming a charge on the state: In the summer, about half of this years new graduates will not find employment That alone will add 200,000 to the newly disposessed. I would hate to go into a decadal cooling phase in our present parlous state! - Which IMO we are, looking at our idling Sun.

I suggest our proposed solutions are in fact now entirely hypothetical: We do not have the money, will power or resources to reverse our course. With luck, we will occasionally have some electricity to watch Top Gear (I hope so - he is an antidote to the remorsless droning on AGW).
Rgds and Good Luck
Dropstone

I seem to recall a figure from Chris Vernon which was that the whole UK transport system could be electrified with adding 20% to the generating capacity. The savings in primary energy use in transportation far outweigh this.

I sense a sombre, approaching desperate mood. After 4 years sabbatical, it looks like I'm about to get a new job - maybe that's why I feel less desperate. One thing for sure is that if we continue to make the wrong choices then industrial civilisation is doomed in the near term.

Well I found your article interesting. The commentary that followed, interesting too, but for different reasons.

Short of going back to horses and wagons, presumably we'll need to continue producing some kind of short to medium range wheeled transport going forward, if only to transport the cargo from clipper ships and local farms to our communal farmers markets. Not everyone has a river nearby, and we can't run rails out to every agricultural area. And horses and wagons have a very limited range.

And as a purely practical matter, the military certainly won't abandon motorized vehicles - even if just for logistics. Well, if YOUR military does, and someone else's military doesn't, and you are in possession of something they want, it probably won't end happily for you.

So given that situation, why not take a look at what the different options of motorized transport are, so we can make an intelligent selection of the most reasonable technology going forward?

Euan, thanks for providing a "back of the envelope" thumbnail analysis.

One thing I'd like to see is an analysis of how much energy it takes to construct a "good" battery, vs a hydrogen tank. My understanding was, good batteries are quite expensive and use some difficult-to-scale resources, while hydrogen (even though it's a more profligate consumer of electricity) you store in a big metal tank which we can already produce en masse. Would that high battery cost issue make a hydrogen car more interesting?

(I also heard the PEM stack was expensive, but there are a lot of projects to make it cheaper)

I don't have any axes to grind or positions to defend, I'm just repeating stuff I heard and curious to hear more from someone more in the know that myself.

You're looking at it backwards, I think. You've said that because some groups will have something, giving them an advantage over group that don't, then all groups must have that thing. Therefore, it will be possible for all groups to have that thing.

This doesn't follow. Natural limits will ensure that consuming resources beyond their renewal rates, however low that consumption, will result in depletion of those resources.

Dear Euan,
the 20% figure may well be true, but add to this the new-build base load to compensate for the Nuclear and Coal power stations due to come offline. We need about 40% of new just to stay the same as we are now and out to about 2015-2018. So overall, the additional 20% is more like 60%. At one time, this was possible, however that time is more or less past and even if a massive build programme were undertaken then it may be too late. Furthermore, due to the recent disaster in our economy, we may find that the future builders of any new component of baseload - especially nuclear - suddenly become reluctant to build them. Since a pauperised nation is unlikely to pay enough back to cover the build and operating costs. - I assume the companies involved will want to make a fair return. So I think it possible that they might not actually get built, along with the Severn Barrage, The 3rd Runway and dare I say, the Western Peripheral Route.

I suspect that we heading for a period of rolling blackouts in the near future and this will probably finish off what is left of UK manufacturing. And, as we move forward, as gas becomes expensive and is piped to countries that can pay, then a few winters like this last one (and I think more are liklier than not) then we are finished.

Assuming that we can get through this chicane, then I agree that electric vehicles are the way to go. With Diesel and Petrol in restricted use for Agriculture /food transport (at least for some time), Emergency Services and the Armed Forces. Movement of goods should be by electrified rail (and sea). But all this takes leadership and vision and action.
Good luck with the new job
dropstone

I've heard a 50% increase would be needed, to compensate for other factors like downtime needed for maintenance. However, the country would then be looking to grow electricity generation from a larger base, to maintain economic growth.

Really, there is a very simply yardstick to use for society. Does it follow the axioms of sustainability, such as those distilled by Richard Heinberg? If not, then it isn't sustainable.

Good luck with the new job but don't let your own personal short term circumstances mask that you're living in an unsustainable society. Electric cars won't make that go away.

It's probably a 50% capacity factor increase and a 20% generating capacity increase, since renewables like wind tend to generate a third of what their nameplate capacity is.

By 2031 I imagine the whole of the worlds fleet of cars will have been replaced. This is likely to happen whether you like it or not. In my opinion, its best to replace the existing car fleet with the most energy efficient vehicles that can be designed and built, and that will include cars that last 15 to 20 years and which can be largely recycled, running on renewable energy. Too bad that you don't agree with me.

Given the ways things are financially, let alone in terms of energy, that's a moot point I'd say and simply speculation. I am in complete agreement with you that if cars ARE to be manufactured then they should be recycled and be as efficient as possible. Where we seem to differ is that you are of the opinion that those cars SHOULD be replaced whereas I'm generally of the opinion that there's better things for us to do with our resources, namely, to satisfy our transport needs in the future with something other than cars. I imagine that there will be car pools, taxis, car clubs and so on, but that the idea of 1 household = 1 (or 2 or 3) cars is over in my view.

It seems you favor some version of a mythical paradise where all the houses, shops and places of work are somehow just magiced into one place so that folks can walk or cycle everywhere.

The future is local and the idea (prevalent only over the last 50, or fewer years), that you just drive 50 miles at the drop of a hat to buy a TV or something, is simply a fossil-fuelled blip in the way humans live and is not sustainable. You seem to imply that the opposite - hypermarket and retail theme park 4 miles out of town and work 30 miles away is something that is destined to continue. I did not say walk & cycle OR private car. What I'm saying is that the current presumption that we all have our own car, or aspire to that ideal, is an idea with no future. Given that we're already using the equivalent of 3 planets to support the current human experiment then we must reduce demand for everything, including travel, and what travel needs do remain should be satisfied by walking, cycling, public transport and probably several types of shared car use.

the global car fleet will be replaced whether you like it or not.

Such certainty! And presumably by extension, the global domestic fridge fleet, global CH boiler fleet, global TV fleet, global computer fleet, global washing machine fleet, global cooker fleet and so on and so on. And it logically follows from that, that all of those in their time must also be replaced by newer, more efficient models. You are obviously in support of the current market paradigm:

Our enormously productive economy demands that we make consumption our way of life...that we convert the buying and use of goods into rituals...that we seek our spiritual satisfaction, or ego satisfaction, in consumption. We need things burned up, worn out, replaced and discarded at an ever-increasing rate.'
Victor Lebow, US Retail Analyst, 1950

This is the same thinking that tells us that we must have a 3rd runway at Heathrow because air travel expansion is such that we will run out of capacity if we don't. Do you support that analysis? Is there a moment of reflection to ask if these scenarios are actually desirable? It's a bit like the 'sexy' attraction of building Zero carbon homes to address our domestic energy use so that's where that attention goes. However, even before the finance/house building crash, the government's own figures say that 70% of 2050's houses have already been built. So, what we need is to reduce energy demand in our existing buildings, not knock them down and replace them with Zero-carbon homes (whatever that means!).

My OND was in automobile engineering and I was obsessed with cars for many years before I realised that we had to find alternatives. I don't watch Top Gear. I haven't had a TV since 2003 and even when I did I only watched about 2 or 3 programs around the time when it started. I've seen a minute or two here or there on the web or at friends' homes. JC's mindset is exactly the kind of world view that has to be ditched if we're to have a real chance of a future. His unbounded arrogance and dismissal of anyone who wants to spoil his fun exemplifies the way of thinking that is sending us down the toilet.

I'm not quite sure what you are trying to say in your comment on cycle lanes, but it is as true in 2009 as it was back (I think) in the time of Thatcher when someone commented that "The UK does not have a transport policy, it has a roads policy". We could probably also insert 'and air travel' before 'policy'. Considering the benefits in personal health, reduction of pollution, emissions and noise engendered by a widespread adoption of the most efficient transport system on the planet, cycling provision is still given almost less than lip service by the government. Just look at the abysmal state of cycle carriage on trains for only one example of the total lack of joined-up (or indeed any) kind of thinking at all for an alternative to the car. Walking and cycling are my main way of getting around, and I'm reminded every single time I use my bike how atrociously I'm treated as a cyclist compared to car users. This is something that absolutely has to change so that people's first thoughts of options on how to get to where they want to go is walk, then cycle, then public transport and then car. Not just car.

Thanks for your replies Euan.

The authors of the following book put the total number of motor vehicles (including trucks) at one billion, projecting two billion within 20 years. Personally, I think that they are nuts. How many billions of barrels of oil would it take to make another billion vehicles?

BTW, regarding 2031, our middle case is that the top five net oil exporters--Saudi Arabia, Russia, Norway, Iran and the UAE--will be collectively approaching zero net oil exports by 2031.

http://www.amazon.com/Two-Billion-Cars-Driving-Sustainability/dp/0195376...
Two Billion Cars: Driving Toward Sustainability

At present, there are roughly a billion motor vehicles in the world. Within twenty years, the number will double to 2 billion, largely a consequence of China's and India's explosive growth. Given that greenhouse gases are already creating havoc with our climate and that violent conflict in oil-rich nations is on the rise, does this mean that matters will only get worse? Or are there hopeful signs that effective, realistic solutions can be found?

In Two Billion Cars, transportation experts Daniel Sperling and Deborah Gordon provide a concise history of America's love affair with cars and an overview of the global oil and auto industries. America is still the leading emissions culprit, and what is especially worrying is that developing nations are becoming car-centric cultures as well. The authors explain how we arrived in this dangerous state, and also what we can do about it. Sperling and Gordon expose the roots of the problem-- the resistant auto-industry, dysfunctional oil markets, short-sighted government policies, and unmotivated consumers. They zero in on reforming our gas-guzzling culture, expanding the search for low-carbon fuels, environment-friendly innovations in transportation planning, and more. Promising advances in both transportation technology and fuel efficiency together with shifts in travel behavior, they suggest, offer us a realistic way out of our predicament.

Ironically, the authors contend that the two places with the most troublesome emissions problems--California and China-- are taking the lead in developing effective strategies that can help wean us from our reliance on conventional, petroleum-fueled cars. California's embrace of eco-friendly policies, which Governor Arnold Schwarzenegger discusses in the foreword, and China's willingness to confront the twin environmental and energy crises wrought by an exponential growth in cars, suggest that if they can develop ingenious and effective solutions, then there really is reason for hope.

Alan Drake's recommendation for electric transportation (San Angelo, Texas, circa 1908):

Electrification of Transportation (Drake)
http://www.energybulletin.net/14492.html

Top Gear: Perhaps Clarkson would like to test drive this model?

haha, depends whether we're talking about the car ;-)

OK, real reason for my post is that I have a question about the power output of electric cars. We are a family of 5 and when our (ICE) car is fully laden it can struggle with steep hills. How would a battery powered car fare with, say, a 1 in 5 upslope when fully laden? What got me thinking about this was when I helped an elderly chap get his electric one-person vehicle (not sure what the technical term for these things is) over some gravel up a very shallow incline.

One of the key properties of oil is power density - how important is this when thinking electric cars? Obviously the car could be made of lighter materials but the load can't (not the people anyway!).

TW

TW,

To answer your question we have to make some basic asumptions so here goes:

Diesel fuel contains 10 kWhr of energy per litre, (or to be precise 45 MJ/kg)

A lead acid battery based on data from the current Yuasa catalogue can store 30 Whr/kg. This is based on a 75 Amp hour deep discharge battery that weighs 20 kg. The reserve capacity is specified at 25 amps for 120 minutes. This equates to roughly 600 Whr. You can see from this that weight for weight, diesel contains 330 times as much energy.

Now look at energy content on a volume for volume basis, the battery I have specified has a volume of 9 litres (probably slightly less to be fair because I have used the case dimensions). This gives about 66 Whr/litre so to allow for my error this can be rounded up to 100 Whr/litre. On a volume for volume basis diesel contains 100 times more energy than the lead acid battery.

Many will argue that the lead acid battery is old technology, and that is true, it is the oldest rechargable battery technology and has survived 150 years of competition, despite its poor performance. The reason for this is simple, infrastructure to manufacture them, low energy required to process lead, plenty of lead from centuries of extraction, all leading to a very low cost battery. Infact one can buy the above battery for about 60 UKP. An automotive equivalent is about 30 UKP.

So lets assume your car body shell is 1000kg and you have 500kg of luggage. I will fit you a lead acid battery weighing 1000kg. Total vehicle weight is 2500kg. Road gradients use the sine gradient, this means 1 in 5 is one unit raised for every unit along the slope (not as many believe along the horizontal).

A 1000 kg lead acid battery contains 30 kWhr (108 MJ) of energy. Using the formula PE=Mgh, we can calculate your car can be raised 4400 metres in the vertical plane. Since you are on a 1/5 slope this is 22 kM or 13 miles added to the mileometer of your car. This assumes no tyre friction, no air resistance and no electrical losses, but gives you an idea of what to expect. There are now batteries that may be 5 times as energy dense than the lead acid on a weight by weight basis, but you can search this information out and do the sums quite easily. This does not necessarily imply electric vehicles will not be useful in the future, but it does ilustrates why the ICE has dominated "free steer" transport, and will do so until there is a fundamental shift to the way we live, whether planned or imposed. Air travel is stuffed I suspect.

If you look at flat level travel, it does not look as bad. By comparing a 55mpg diesel car, we can assume that such a vehicle uses about 800 Whr/per mile. If an electric car can half this to say 400 Whr/mile, due to increased efficiency then you would have a range of 75 miles. I think some on this website claim 200 Whr/mile so with some luck you may exceed 150 miles. This is probably ok for most people's daily needs, but I would have to change my job. The problem is the inconvenience of the charging process, which would require more discipline than refuelling an ICE. Again this may be something we have to live with, but at present it would deter most folk from driving electric. As I said above there is considerable road side parking in the UK because many people don't have driveways. The infrastructure is not in place to accomodate roadside charging.

The torque/speed characteristics of electric motors are superior to the ICE. A standard idustrial induction motor can produce 3 times its rated torque for short periods (more if you drive the motor magnetic circuit into to deep saturation). An ICE has a maximum torque quoted at a particular speed and that's your lot. From a hill start point of view, providing the power converter (inverter) is suitably rated, hill starts will be easy. The efficiency of the drive system will be low during this high torque low speed situation, but will improve as you accelerate. Your range will ultimately suffer if you stop and start too often, but that is true for all "free steer" vehicles. I worked on battery mining locomotives (rail guided) that used series wound dc motors. These produced maximum torque at low speeds and could spin the wheels, we ended up putting rubber tyres on the steel wheels!. Range and charging times was the problem then, as it is now. Most mines had at least one spare battery per loco and it was not unusual for diesels to have to tow them in.

I think my numbers are correct, though I'm sure if I have a decimal point in the wrong place someone will take great pleasure correcting me.

I believe some entreupeneur has either began a battery swap business or has plans for it on Ohau, HI. It is a pilot project in a place where the batteries really can't go anywhere, but this is one way to attack the range/charging facilities problems. I couldn't come up with an article but will keep searching.

Partypooper,

"The problem is the inconvenience of the charging process, which would require more discipline than refuelling an ICE. Again this may be something we have to live with, but at present it would deter most folk from driving electric"

Most Canadian's plug in their car's block heater and battery warmer during winter, at home, at work and at shopping centers. It's a slight inconvenience, but much less than a car that won't start at -30C. A lot of women would consider filling up a ICE with petrol much more of an inconvenience. I suppose you don't make tea or toast at home because it's so inconvenient to have to plug in these appliances once a day.

If you had a PHEV, and could save 6 gallons diesel per day( if you actually drive 150miles too and from work) I think you would find it more convenient to plug in at home and at work.
The issue of many people not having back lanes or garages or parking places is a big issue but Canadian's seem to manage by trailing extension cords across the snow. The early adopters will be the suburban drivers who live too far from public transport, but have garages on on site parking spaces.

Neil, I think you just reply, without reading what I am saying. I'm not getting into another pointless argument with you. Why do you not have mass electric car transport in Canada then? It sounds a good place to have started developing them years ago. Don't you have a General Motors or Chrysler plant out there?
Because I work from home and travel to site I can go any where in the uk. Its not unusual for me to to 3000 miles a month, some days I do none at all, but I often do 360 miles in a day, say from where I live to the North east. As I said my job will become impracticle and I can join the ever increasing UK dole queue. Perhaps the uk government should divert all our unemployed to electric car infrastructure building, I might then share your optimism.

I said cars parked on the roadside, impling trailing extension leads across the public highway. Snow or tarmac, it would not be permitted since its a tripping hazard. Again, you need to read what I say and reply accordingly, not just pick out words from my post and invent a reply that suits your argument.

Partypooper, I think you are overreacting to Neil's post. This may be because of previous experience with him, but I don't think (if that's the case) that it's justified this time.

Putting that on one side, I think your original post is very helpful and useful - it certainly illustrates the new constraints that we will suffer if/when we lose oil as a fuel for road transport. And this change seems somewhat inevitable unless someone comes up with a perpetual motion machine that actually works (which may of course happen).

But if the future is as you suggest then we will be forced to cut our cloth accordingly, and I have been thinking for a while now that jobs which involve travelling long distances will have to change - what will probably happen is that people's work will become more localised. SO rather than a plumber doing work over a radius of maybe 50 miles, alongside lots of other plumbers all doing the same thing, what will happen is that each of them will start doing their work much closer to home, maybe within a 5 mile radius. The total number of plumbers will stay the same, it's just that they won't all be travelling long distances to do their work. This seems somewhat inevitable unless we end up with some new and currently undiscovered source of limitless energy.

If your job cannot change in this way then it will have to change in some other way, such as using public transport for the long-distance segment of the journey, and if the amount of time spent travelling increases to the degree that you can do less jobs per week then the cost for the job will go up. The whole industry will face the same problems, after all!

nc,

I don't disagree with you at all, may be its just a cultural and demographic issue. The problem with companies that employ highly trained service staff is the trade off between employing a few people and forcing them to travel the entire country or employing more around the country at greater "standby cost". There are no easy answers. I hate driving but enjoy my job. There are 4 people to cover the country, there used to be 5 but one left and was never replaced due to cost saving. This is just the opposite to what is required.
To be honest, I can see the problems, but not easy solutions.

I said cars parked on the roadside, impling trailing extension leads across the public highway. Snow or tarmac, it would not be permitted since its a tripping hazard.

I don't quite get it. Whats the concern with trailing an extension cord from your home out to the curbside to plug in a vehicle or two? Very common in Canada.

Someone would trip over it and sue you, it would obstruct folk with pushchairs as well. The local council would soon kick your ass (if ass is the Canadian term, its arse in the uk). Elf un Safety as we call it.

I suppose, if people such as you who seem determined not to have any electric vehicles about, begin to sue their neighbours for really stupid @#$%, then perhaps someone will need to come up with a system of burying the cable under any intervening sidewalks, or perhaps smart plug-meters mounted on parking meters.

But man, talk about determined to have problems.

Thx for the comprehensive reply PP. I guess the bottom line is that power equivalence (or better) is achieved by EV's compared to ICE's by having a much bigger and heavier battery, compared to diesel weight in an ICE, to compensate the lower energy and power density. Obvious really, if only my basic physics wasn't as rusty as a 1970's off-shore oil-rig!

This little beauty certainly doesn't seem to suffer from lack of bhp;

http://www.dailymail.co.uk/sciencetech/article-475542/The-ultimate-elect...

Just wait for mass production to bring the price down by an order of magnitude. Not sure where I'd put the kids mind you - leave them at home I guess ;-)

Not that I'm suggesting this car is a good idea in a world where we should be consuming less, walking/cycling everwhere and just kinda making do. But it does look nice. And I bet it would be fun to drive. No! stop it! Oh dear, I'm really not sure I'm cut out for a post-apocalyptic low-energy world.....

TW

In terms of your numbers, I think the cheapest(per kWh stored) suitable battery tech would be the best bet for comparison, not lead acid. We are after all looking at consumer products, not industry. If we were all hell bent on using the same as industry all autos would be fitted with 8-10 speed manual transmissions and 10-20kWh engines so they could have close to the efficiency per ton that tractor trailers average. Heck, you could use Nickel Iron if you wanted to make the comparison look even worse. ;)

The most suitable chemistry in terms of weight/volume per unit energy, cost per kWh stored, lifespan, and power per weight/volume (LiFePO4) seems to be at ~20lbs/kWh, so 1000kg of battery would result in ~110kWh of energy, which would take a compact EV ~450 miles, and the pack would take up ~28ft^3. A 10 gallon tank takes ~1.34ft^3, so we're looking at ~21 times less energy density, and ~28 times greater mass per distance traveled.

A 55mpg diesel is actually at 889Wh/mile assuming a man sized gallon, so lets say ~900Wh/mile. The EV1 was at ~250Wh/mile at the plug, with lead acid batteries at ~80% efficiency. The newer Lithium chemistries, specifically LiFePO4, and possibly others, are at 95+%, so a current version would almost certainly be at ~200-250Wh/mile. At high torque we may see a ten percent drop in efficiency, but this isn't exactly a deal killer.

One significant advantage with electrics, especially using a suitable chemistry, is energy recapture when braking, which can cut down on both maintenance and energy consumption. New chemistries can take up to ten times more energy during a charge than what is seen in current HEVs (NiMH), which significantly increases energy capture. Granted, getting on the brakes is never the best way to drive efficiently, but getting back half that energy significantly decreases the energy consumption per mile.

Hi Ralf we meet again!

A 55mpg diesel is actually at 889Wh/mile assuming a man sized gallon, so lets say ~900Wh/mile

I think I quoted 800 Whr/mile in my example so if it makes you happier use 900, it makes little difference to my point above if that's what you are referring to.

I will tell you how I got to 800Whr/l though.

Diesel 45MJ/kg, density 0.8, giving a nice convenient 10kWhr/litre. This is near enough for all practical purposes, unless you are desperate to gain an advantage over me.

55 mpg is 55m per 4.54 litre = 12 miles/litre or per 10kWh. 10/12=833 Whr/litre. So round up if you want, but most mathematically minded would round down, as I did.

The argument is so clear for all to see, Diesel has a massive advantage over any battery technology you chose to quote, or speculate may arrive by the application of hope. This is the reason why battery vehicles will become a solution of last resort not a natural choice by the consumer.

I would not be suprised if private transport is vastly scaled down in the future, once the consequences of peak oil and expensive transport fuels bite hard.

You can't make wind turbine blades from electricity, you need oil as a raw material (epoxy resins)
You can't make car tyres, paint, GRP body shells, seats etc etc from electricity. The entire supply chain will be hit by oil price. If the answer was so easy, some one would have made the change by now.

Partypooper,
"You can't make wind turbine blades from electricity, you need oil as a raw material (epoxy resins)
You can't make car tyres, paint, GRP body shells, seats etc etc from electricity".

I think you mean to say that many of these compounds are "usually" made from oil or natural gas( especially plastics, synthetic rubber). Of course rubber is also made from latex, derived from rubber trees. Most paints are made from vegetable oils or latex. A basic feedstock for plastics is ethylene, which can be generated from reduction of ethanol.

Epoxy resins are derived from phenol, which is made from benzene. For some applications Benzene is synthesized from acetylene, derived from CaC2, produced in an electric arc furnace from limestone and any source of carbon.

You are correct that oil prices rises will influence the prices of these chemicals, but epoxy prices are only a small part of wind turbine input costs, 85% of a wind turbine is steel, cement costs for foundations are also relevant.
The oil input into steel is minor( transport). Electric arc furnaces are used to re-cycle steel. The annual junked cars in US would provide all the steel required to build X50 more than the 20,000 large wind turbines build world-wide last year

I wonder how much of the rubber used in road vehicle tyres comes from trees? I have seen it quoted it takes 6 gallons (UK ? or US ?)of crude oil to manufacture a typical car tyre, though that seems high to me.

For some applications Benzene is synthesized from acetylene, derived from CaC2, produced in an electric arc furnace from limestone and any source of carbon.

Not very energy intensive then, I take it. I see EROEI falling a bit here!! Wind turbine supplying an arc furnace to make calcium carbide to make epoxide, the plot thickens.

Granted you could synthesise everything if required, that I realise. Its the cost of doing it that counts when the luxury of cheap oil ends. The point here is really whether it will be done, not that it can't be done. For that you no more know the answer than I do.

The chemical synthesis industry is very flexible in feed-stocks, oil, NG, coal, plant products, whatever is cheapest. All tires and other rubber was from natural rubber trees prior to 1942when Japan captured the SE Asian plantations. Other plants can also produce latex. Natural rubber is still used for large aircraft tires, heavy machinery, large trucks etc the things we really need to keep industry going.
You are probably correct that car tires are going to become more expensive when NG is no longer available, but we are a long way from running out of NG for chemical feed-stocks, we are still BURNING IT! Look forward to seeing tire re-treads again on future EV's.
Judging from how people in WWII treasured old tires, the price of latex can go up considerably and people will still pay the price to be able to drive the car.

Neil,

You mention retreads (remoulds), I always used to use them, but cheap imports have killed the industry in the UK. Re moulds had a bad name, but it was largely a myth. Its an irony of this wasteful world, where we have millions of tyre carcases that we just granulate and ship the bits that don't separate (steel cord/rubber) properly in the granulator to china. Thats capitalisation and the free market for you.

The point is everything is going to get more expensive. In fact diesel is on the rise again although oil is still relatively cheap. I'm not sure why, is it to do with the grades of crude oil not being suitable for diesel production?

Not only are we still burning nat gas, we are building more powerstations to burn more of it. Lets hope there's is plenty of it.

I agree that oil and NG are going to get more expensive, and products made from OR competing with these will also become more expensive. On the other hand, many inputs for non FF energy, steel, cement,copper, resins, glass, are going to be cheaper while most of the world is in recession( China excepted). This may not make electrical energy cheaper, but renewables more profitable, allowing continued rapid growth perhaps even without government support.
For private EV and PHEV vehicle transportation, the initial cost will be higher, but operating costs will be lower, even allowing for expensive tire replacements.

Diesel prices may be going up because of China's continued imports, its economy is still growing judging from recoveries in iron ore spot prices. China seems to be buying up resource companies now while prices are low, setting themselves up for when the recession/depression is over.

I think I quoted 800 Whr/mile in my example so if it makes you happier use 900, it makes little difference to my point above if that's what you are referring to.

I will tell you how I got to 800Whr/l though.

Diesel 45MJ/kg, density 0.8, giving a nice convenient 10kWhr/litre. This is near enough for all practical purposes, unless you are desperate to gain an advantage over me.

55 mpg is 55m per 4.54 litre = 12 miles/litre or per 10kWh. 10/12=833 Whr/litre. So round up if you want, but most mathematically minded would round down, as I did.

Your compounding errors by going the long way around so to speak. If you simply went to an online conversion site you would find that a (man sized) gallon of distilate #2 fuel oil has 48.88kWh of energy. At 55mpg, that would be 48,880Wh/55miles=889Wh/mile, or ~900Wh/mile. Being mathematically minded like you mentioned, we would end up rounding up to 900Wh/mile, not down to 800Wh/mile. Also, given that your estimate of EV energy consumption at the plug was closer to a SUV than a compact car, we're looking at some pretty big errors, with the compound result being off by ~100% or so.

The argument is so clear for all to see, Diesel has a massive advantage over any battery technology you chose to quote, or speculate may arrive by the application of hope. This is the reason why battery vehicles will become a solution of last resort not a natural choice by the consumer.

Diesel has the advantage in terms of the density and volume of the onboard energy. Electricity from renewables has the advantage in terms of efficiency, instantaneous power, pollution, other externalized costs, location (the ME isn't exactly a stale place), cost per mile, and sustainability. While oil will eventually peak and decline, the sun will continue to shine and the winds continue to blow (among other things) for a while longer. In terms of what's a "natural choice" by the consumer, there's no such thing. People will choose whatever they will choose at any given point in time depending a variety of influences, including cost, range, power, and so on.

You can't make wind turbine blades from electricity, you need oil as a raw material (epoxy resins)
You can't make car tyres, paint, GRP body shells, seats etc etc from electricity. The entire supply chain will be hit by oil price. If the answer was so easy, some one would have made the change by now.

There's no point in changing now. We're producing the most oil we've ever produced over the last few years. As supply dwindles, then we'll see replacement and changes. In terms of epoxy, while FFs do present cheap feedstocks, it could be synthesized from renewable sources like most other things. Rubber for tyres, believe it or not, can be natural or synthetic, as can other products that have been dominated by FF feedstocks. Of course we aren't going to be turning electricity in directly into a car tyre, however we can an probably will use the variety of renewable energy sources and renewable feedstocks in order to produce similar materials as the supply of our current feedstocks dwindles.

I think I gave a range of values from 400Wh/mile to 200 Wh/mile, if memory serves me correctly. It would save so much time if you read my posts properly.

VIZ

If an electric car can half this to say 400 Whr/mile, due to increased efficiency then you would have a range of 75 miles. I think some on this website claim 200 Whr/mile so with some luck you may exceed 150 miles

10700 Wh/l is my official figure for diesel, 10000 is a nice round number and is near enough. As I said i am happy for you to use 900, I just explained why I got 800 or (825). The whole post was to give an illustration, and I'm sure the guy who's post i responded to would be capable of modifying the numbers accordingly for various technologies. I seem to recall suggesting a 5 fold reduction in battery weight may be possible. I used lead acid because real data is readily avaiable and there are set standards for their ratings. I have a Yuasa catalogue with all the data. Euan suggested electric cars were twice as efficient as ICE so I used 400 (800/2), I then used the figure (you may have quoted on a differnrt post) of 200. Can't be fairer than that.

I think I gave a range of values from 400Wh/mile to 200 Wh/mile, if memory serves me correctly. It would save so much time if you read my posts properly.

You used 400Wh/mile in your comparison which is what decade and a half old electric SUVs pulled. 200Wh/mile given a small car doesn't require luck since that's was what we saw a decade and a half ago with relatively lossy lead acid batteries. I suggest seeing a doctor about your memory. ;)

Heck, even as a range that's off since ~100Wh/mile seems to be the lower end.

By comparing a 55mpg diesel car, we can assume that such a vehicle uses about 800 Whr/per mile. If an electric car can half this to say 400 Whr/mile, due to increased efficiency then you would have a range of 75 miles.

I have a Yuasa catalogue with all the data. Euan suggested electric cars were twice as efficient as ICE so I used 400 (800/2), I then used the figure (you may have quoted on a differnrt post) of 200. Can't be fairer than that.

Of course you can be fairer than that. Assuming contemporary battery chemistries, correct energy consumption (closer to 200Wh/mile than 400 Wh/mile for a small car), and so on... Furthermore, if looking at the difference between peak and average EV efficiency, do the same with conventional vehicles. If you look at modern examples and apply the same standards to both sides of the coin, so to speak, then you can say you've been fair.

I think you've lost the plot.

I think you should think less about plots and more about accurate, up to date figures and consistent comparisons. ;)

Repeat, I think you have lost the plot mate. if all you can pull me up on is an approximation error of 12% then you are clutching at straws.

Hi again Roflwoffle,

Looking at the MIEV (pretty new technology by all accounts) it has a claimed range of 80 miles using a 16kWh battery, thats 200 Whr/mile by my reconing. If you read the test, and favourable report (by Autocar I think), 50 miles is a more realistic range and thats 320 kWh/mile. Considering my estimates were more guestimates based on experience I was remarkably close to reality and luck would indeed be required to achieve the former value. (The GWIZ appears to have 240Whr/mile, but as reports again state, the 40/48 mile range is a conditional maximum). My diesel will do 70MPG at 56mph, though I would not make any claim that this is realistically achievable under normal driving conditions. Same goes for EV's, its called sales pitch.

There are two charging rates specified, 15 amp at 200 volt for 7 hours, thats 21kWh so 16/21 gives 76% efficiency.
The 50kW 1/2 hour charge (to only 80% capacity) gives (16*0.8)/25=51% efficiency. Now we are getting more realistic figures, not optimists dreams. I suspect the 20 GW grid capacity estimated for the uk to support all electric transport may be closer to 30GW, allowing for over optimism.

Still, not a bad car, at 25,000 quid it will be affordable by the rich city citizens and is ideal for say London. As a real contender to the ICE, then there's work to de done. Perhaps not too much though, a range increase of 6 fold would be useful and a 50% reduction in cost might help make it affordable to normal people. the world all electric by 2030, I doubt it, 2050 maybe assuming our vehicle manufacturing base suvives this horific financial mess and the power grid expansion takes place.

Looking at the MIEV (pretty new technology by all accounts) it has a claimed range of 80 miles using a 16kWh battery, thats 200 Whr/mile by my reconing. If you read the test, and favourable report (by Autocar I think), 50 miles is a more realistic range and thats 320 kWh/mile. Considering my estimates were more guestimates based on experience I was remarkably close to reality and luck would indeed be required to achieve the former value. (The GWIZ appears to have 240Whr/mile, but as reports again state, the 40/48 mile range is a conditional maximum).

This is the only report of a test drive from Autocar I've found while searching and it makes no mention of range. According to anecdotal accounts larger conversions like the AC Propulsion Ebox are around 250Wh/mile. That said, even at 300Wh/mile electric and 900Wh/mile conventional compared to 400Wh/mile and 800Wh/mile respectively you've gone from three times the energy consumption to two times the energy consumption, a far cry from a 12% difference.

There are two charging rates specified, 15 amp at 200 volt for 7 hours, thats 21kWh so 16/21 gives 76% efficiency.
The 50kW 1/2 hour charge (to only 80% capacity) gives (16*0.8)/25=51% efficiency.

In terms of the Mitsubishi's charging efficiency, you can't possibly know that w/o more data. Lithium based chemistries are charged at some specified current up to some voltage, then held that voltage while the current is dropped to some point, usually around 10% or so of it's maximum value. At the peak charging current, the battery will only get on average a bit more than half of it's charge over a third of the time, and then the current will drop as the battery is charged to full capacity over the other 2/3rds (roughly) of the charging period. It's not 15A@200V for 7 hours, more like 15A@200V for a bit more than two hours and then the current drops while voltage is held constant for the next four plus hours.

Now we are getting more realistic figures, not optimists dreams. I suspect the 20 GW grid capacity estimated for the uk to support all electric transport may be closer to 30GW, allowing for over optimism.

Well, at least fabricating more data for your optimistic one side dream anyway. ;)

There's no way you can know what the charging efficiency is w/o having a record of voltage/current over time, and like I said before, you can't simply assume the batteries are charged at the maximal current/voltage all the way through. That isn't how Lithium based chemistries work.

If you want to run a far comparison, compare a group of EVs from large car makers on average to a group similarly sized (petrol and diesel) conventional vehicles from large car makers, not using an EV that consumes about twice what the most similarly sized EVs do and charges at some constant rate, which isn't how lithium batteries are charged btw, compared to a fairly efficient diesel.

P.S. I can get ~60-90mpUKg at a 40-55mph average speed (stops and all that included) in my MK1 Golf diesel, but that doesn't mean everyone can. ;)

Waffle!

12% error was (as you must know unless you choose not to) referring to my 800Whr VS your 900Whr and as stated is neither here nor there and made no difference to the point, thats why I let it go without a fuss.

If you want other comparable percentages try 350/400 or 200/250. You get very mixed up at times

It's not 15A@200V for 7 hours, more like 15A@200V for a bit more than two hours and then the current drops while voltage is held constant for the next four plus hours.

15A for 2 hours at 200 volts is 6kWh, So 10kWh of energy for nothing then! It gets better by the minute.

Mitsubishi quotes a range of 80-100 miles if the i-MiEV is driven economically

This is quoted from the article from your link, note the "IF". If I drive my ICE economically I will get 70 MPG plus, thats a range of 700 miles for a "five minute charge". The article is from December 2008. Read their current issue and you will see the 50 miles I refer to. I had the magazine physically in my hands, so not from the web. Happy reading.

12% error was (as you must know unless you choose not to) referring to my 800Whr VS your 900Whr and as stated is neither here nor there and made no difference to the point, thats why I let it go without a fuss.

The factor of two compared to factor or three error was (as you must know unless you choose not to) referring to the entire comparison, not just your errors regarding diesel energy consumption. You errors regarding small EV energy consumption were also included. Errors can compound, and given your assumptions being off by a factor isn't exactly a small difference.

15A for 2 hours at 200 volts is 6kWh, So 10kWh of energy for nothing then! It gets better by the minute.

You're really having trouble with this whole lithium charging concept aren't you? As hard as it may be to understand, the current does not drop to zero after two hours and change in this case, but instead drops as voltage is kept constant, to somewhere around 10% of it's maximum value. It's probably something like 15A@200V for 2-3 hours totaling 7.5kWh, then an average of 10A@200V for 4.5 hours totaling 9kWh. We don't know exactly w/o data so there could be more or less energy during the constant current portion of the charge and so on, but that's how lithium chemistries are charged believe it or not...

This is quoted from the article from your link, note the "IF". If I drive my ICE economically I will get 70 MPG plus, thats a range of 700 miles for a "five minute charge".

If I drive my diesel I can too, but most cars aren't diesels and most people don't drive economically, so if you're going to assume real world driving conditions and cars then you're aren't going to focus on just the most efficient conventional model driven efficiently compared an electric, you're going to look at the average of electric offerings compared to the average of conventional offerings give consistent testing conditions, probably standardized fuel economy testing or similar.

Read their current issue and you will see the 50 miles I refer to. I had the magazine physically in my hands, so not from the web. Happy reading.

Scan it and post it up! The only real world tests I'm aware of were between ~160Wh/mile and ~250Wh/mile.

Course, for $12,500 in Japan according to the article, and somewhere around $16500 in the states, it's probably the cheapest new vehicle to own, even with $2/gallon petrol, given the reduction in maintenance costs, and with higher petrol prices, it's a no brainer. I don't think anyone here is saying that electrics are going to be viable for everyone, since they're mostly for people who have fairly short (<30-50 miles, or 50-100 miles if they can charge at work) consistent commutes, just that they use a heckuva lot less energy all things being equal, can be powered by local renewables as opposed to oil from countries who are less than friendly, and cost less to operate too.

you're going to look at the average of electric offerings compared to the average of conventional offerings give consistent testing conditions

Exactly, thats why Auto Car suggest 50 miles is more realistic than 80 and why 55 mpg is more realistic than 70 mpg and also why I used 55 in my comparison.

You're really having trouble with this whole lithium charging concept aren't you?

Not really, no more trouble than you are accepting the obvious reason why the ICE has and still is so successful and will continue to be until it is either outlawed (due to prohibitively stringent emissions regulations)or oil becomes too expensive. By that time our economies may not hold together in such a way as to construct an EV infrastructure and the cost of food will be so problamatic cars may no longer be an affordable option. Infact there is no guarantee private transport has a long term future in the form we have today, ICE, electric or otherwise. There are applications for diesel/battery electric vehicles that are blindingly obvious, City buses for example, but we still use the ICE for these despite pollution issues. There has to be more to this than meets the eye, I suspect cost. London are moving towards this, but its slow going.

I do not have a copy of Auto Car, but it will be the March issue 2009, available in the UK from WH Smiths, which is where I saw it. It has some interesting (and realistic) articles on the new Prius (and others) as well. It very sensibly says that a Prius is not for motorways, and I think Toyota state this, or similar, as well. Its also curious (interesting even) why Toyota are continuing with NiMH technology in the all new Prius, apparently cautious about Li ion for both safety and long term reliability.

I would not go as far as to say most cars are diesel, but in Europe a very high percentage are. Diesels exceeded 50% market share some time ago in the UK, and we are behind Europe on the uptake. I think you (in Canada ?) will benefit from hybrids much more than we Europeans because we already drive economical cars and have done for years, due to fuel taxation. The goverments will almost certainly have to tax electric cars if/when they displace revenue from ICE vehicles. The roads will have to be maintained somehow. Tax relief on different fuel types have always been erroded once the takeup starts.

Hopefully other posters have answered your query. In the first instance, I think the auto industry should be aiming for small electric vehicles with niche commuting around town in mind.

I'm not sure all electric vehicles will ever be practical for that big family vacation. Perhaps for those occasions you hire a plug in hybrid?

Hey Euan, why not just put wind turbines on top of cars to recharge the batteries as you drive down the road? Has anyone thought of this idea and tried it?

Its actually more effective to have an electric motor drive the front wheels and have dynamos driven by the rear wheels to recharge the batteries and reactive breaking to make up for the friction losses in the system.

The wind generator on the roof will cost you more energy because of the additional drag than it can produce. The generator on the rear wheels will also be an energy loser.

Guaranteed loser, every time. This is because the energy to drive those things ultimately comes from the battery, and there are inevitable losses in conversion.

How can it be a loser? No different than an alternator on a gas engine.
It takes very little to turn a generator on a wheel.

It takes quite a lot to turn a car alternator, just turn on the lights and listen to the engine. A car alternator only has an output of a few hundred watts.

In any case if what you suggest could be done, you have perpetual motion, our energy crisis would be solved, the Oil Drum Could close down and we could all go home.

Whoever it is above joking about putting wind generators on tops of electric cars (agreed, that's pretty dumb) should consider moving the wind generators down in front of the vehicle so that they constitute the main frontal area comprising the auto's drag. Here's an interesting problem. What would be the potential power generation of a set of ideally shaped wind generators comprising the 2.348 sq meters frontal area of a 2008 Chevy HHR? http://media.gm.com/eur/chevrolet/en/download/td/td_hhr_en.pdf What would be the effect on the drag co-efficient of the vehicle if these wind generators were placed directly in front of the vehicle facing the direction of normal travel?

This group, in Hong Kong, sells a small wind turbine where each one is so small that it requires 20 of them to make up 1 sq meter of frontal area.
http://www.motorwavegroup.com/files/motorwindhkuarchitectpresentation.pdf
At wind speed of 19 meters/sec, (40 mph) the 1 sq meter of turbines will generate 800 watts. So theoretically, the 2.348 sq meter frontal area of the HHR could generate 1.878 kW with the vehicle travelling at 40 mph without increasing the vehicle drag co-efficient at all. WHat the vehicle actually sees is a "bubble" of ai travelling at reduced speed relative to the vehicle.

I know, it can't really work (as well as above), but it makes an interesting problem to keep widgetheads occupied during a party explaining why ;<]

Engineers have names for wind-catching things on the tops of cars and generators attached to wheels.  We call them "brakes".

or perpetual motion! (sarcasm)

Damn, I hate to have to do this again...but apparently it never sinks in, so here goes...

At last count I saw, roughly 10 percent of world oil production goes into fueling American cars...so add in the U.K. and you get what, 13% at most to fuel cars in the U.S. and U.K. I concentrate on those because we cannot, no matter how much we may wish we could, order the rest of the world not to use cars or to use only the type that we wish they would, and they are generally smaller less powerful cars than ours anyway...

Now if we accept the projections of M. King Hubbert, Colin Campbell, Matthew Simmons, Ken Deffeyes, Westexas on TOD and others who project a declining oil production and as importantly a declining world oil export market that will easily exceed 10% declines in coming years we have to face the fact that what we drive matters not one whit to the timing nor the eventual assured occurance of peak oil. We could STOP driving every vehicle and still be in deep trouble and collapse as world economies.

The U.S., U.K., Western Europe and Japan are already declining rapidly as a percentage of world oil consumption. Our clout in determining the outcome of oil markets, global warming, oil production and oil distribution is marginal in comparison to the scale of world oil markets and declining with each minute. It is very possible that the old rich developed nations have already gone past peak oil consumption, and this is even before the next wave of technical advances which will further reduce oil consumption hit the market.

For advanced nations, the oil age, that being defined as the period in which oil was the primary fuel of growth and change in the world is already behind us. This was foreseen many years ago. It had to happen and it is happening. It has actually taken a bit longer than expected.

(Brief aside...can someone explain to me how the world auto population is going to double if all of the auto manufacturers are bankrupt? Just asking...)

RC

At last count I saw, roughly 10 percent of world oil production goes into fueling American cars...so add in the U.K. and you get what, 13% at most to fuel cars in the U.S. and U.K. I concentrate on those because we cannot, no matter how much we may wish we could, order the rest of the world not to use cars or to use only the type that we wish they would, and they are generally smaller less powerful cars than ours anyway...

If I compare to China, I see that China already has a domestically manufactured plugin, supposedly priced in the low 20's. Whether this car has any quality remains to be seen. India, also is pusuing electric vehicles. I am tending towards the opposite viewpoint, the developing economies haven't incorporated the sense on consumer entitlement that is common in the OECD, and may well lead the way towards less wasteful forms of transport.

The energy density for lead acid batteries is usually given as 0.1 MJ/kg, making diesel some 450 times more energy dense. Fully developed lithium ion batteries coupled to ultracapacitors might improve on lead acid several fold but that improvement is likely to plateau. I think the electric advantage is in drive trains not batteries and the ability to store intermittent wind and solar power.

I also question whether electric vehicles will be affordable in an era of double digit unemployment and whether they will have enough all-electric range for commuting and freight transport. There is also the huge sunk cost in ICE vehicles, many of which remain unsold in car dealers yards. Therefore an option that could be explored is hydrogenated synfuels, using renewable or nuclear hydrogen combined with organic carbon which recycles within the biosphere. The problems are that this field is in its infancy and it may never approach the efficiency of EVs. Then again it may keep some of the 800 million ICE vehicles on the road rather than wasting all that sunk cost. Synfuel may also provide the range needed for commuting and freight.

The energy density for lead acid batteries is usually given as 0.1 MJ/kg, making diesel some 450 times more energy dense.

I think the big issue is that hugher energy density batteries, with proven field lifetimes and safety are going to take a couple of decades to develope. Decent range is currently acheivable with Lithium Ion, teh Tesla Roadster'sclaimed range is 356KM. But the pricetag of this vehicle is over $100K, and IIRC the expected lifetime of the batteries (the most expensive component of the vehicle) is only about four years. A reasonably priced electric, the Aptera is almost ready for the market. This vehicle achieves a reasonable price of $27K, by being both small, and extremely aerodynamic. The real questionmark about such cars is how safe they are sharing the road with much larger cars. Supposedly Zinc air batteries have over twice the energy density of Lithium Ion. But these are at an earlier development phase.

Boof,

I was working in litres for volume and for easy calculation, I just assumed 10 kWhr/litre, its near enough. I should have divided 100kWhr by 0.8 to give 125, but I over looked that. Anyway it illustrates quite clearly why the ICE wins hands down, despite having 50 years less development time than battery technology.

"Anyway it illustrates quite clearly why the ICE wins hands down,"

That would be true if the contest was 'which vehicle has the highest power density?"
If the average driver travels less than 30 miles per trip, and can re-charge almost anywhere a standard electric outlet is available, being able to drive 500 miles on a tank of gasoline, whether an ICE or PHEV is not very relevant. On the other hand if you own an SUV mainly to travel off road away from electricity and service stations then fuel density is an issue.

In the future, the question may become; which vehicle can still operate post-peak oil, or which is the cheaper to operate if gasoline is $20 a gallon?

being able to drive 500 miles on a tank of gasoline, whether an ICE or PHEV is not very relevant.

Audi's marketing department think it is and I would suspect they have done a little market research on this issue. Their latest advert is based on claiming a 700 mile range. Perhaps you should advise them otherwise and save them considerable money.

You can't change history, the ICE has won ans still is winning hands down. The auto industry is now bust, It may never be able to change direction, I have kept my push bike so I can cycle to my local farm shop. I live near a fresh water stream also.

Oh brother.
EROI is always way over 100% efficient!

Try this.
eo/EROEI=einv.

If efficiency is energy out/einv so
(eo+einv)/einv=(eo+eo/EROEI)/eo/EROEI
therefore efficiency=(1+EROEI)x100%

or if you wish for some unknown reason to leave off the energy invested then efficiency =EROEI x 100%

So assuming your values are correct(which I don't believe they are) wind the efficiency would be (20+1)x .9 x.97 x.92=16.82 (1682% efficient)
For your gasoline(which looks wrong)
(30+1)x .9 x.4 = 11.16 or 1160% efficient.
For your hydrogen starting with wind
it would be (20+1)x .24(Bossel)=5.04
or 504% efficient.
For corn ethanol it would be
(1.5+1) x .95(?) x .4 = .95 or 95%
but the efficiency of ethanol using stalks for fuel would be closer to
(4+1) x .95 x.4= 1.9 or 190%.

I could say much more but why bother?
The mad idee fixe of EROEI is a Gospel here.

If we use ethanol as an example, the main energy input is nat gas and gasoline / diesel. One option is to simply put the nat gas (liquefied) straight into a car and this would provide an energy efficiency similar to the gasoline ICE. Opting instead to convert the nat gas to fertilizer, drive the fertilizer to and spread it on a field, harvest, transport and refine the corn all uses energy that is barely recouped by the solar energy captured by the growing part of the cycle.

Your efficiency calculations >>100% smack of getting something for nothing.

If we use ethanol as an example, the main energy input is nat gas and gasoline / diesel. One option is to simply put the nat gas (liquefied) straight into a car and this would provide an energy efficiency similar to the gasoline ICE. Opting instead to convert the nat gas to fertilizer, drive the fertilizer to and spread it on a field, harvest, transport and refine the corn all uses energy that is barely recouped by the solar energy captured by the growing part of the cycle.

Actually no.
Ethanol is net energy positive(except to Pimentel); for every BTU of natural gas you get 1.3 BTUs of ethanol. Liquified natural gas is net energy negative--for every BTU of natural gas you get .8 BTU of liquified natural gas.

Another big error is the idea that you can run battery powered cars on 100% renewable energy from the grid.
There is no such animal; the best you can do is 20% renewable energy to the grid as beyond that point the grid becomes unstable, unusable.

Well, then you have to add in electricity from fossil fuels at 33% efficiency for the remaining 80% of the grid. So input 2.4 BTU of natural gas input to the power plant would turn into .8 of a BTU of fossil electricity to be added to .2 BTU of 'renewable' electricity to charge the battery of that car at 75% (rectifier).
(eo=.75 BTU)/(einv=2.4 BTU of gas) is an EROEI of .312 which is horrible, horrible!

I agree - Euan's formulae are at fault here. Efficiency is not (EROEI-1)/EROEI, but EROEI. The efficiency of an oil well is 3000%, the efficiency of a wind turbine is 2000%, the efficiency of an ICE is 30%. You are getting something for nothing, or more accurately you are getting more for less - this is the origin of economic growth. The point is that in 10 years the efficiency of an oil well will have declined whereas the wind turbine will be 2000% as long as the wind blows.

Efficiency in your bar graphs is apples to oranges. The thermal efficiency of a heat engine will always be low because mechanical energy is more valuable/lower entropy than heat energy. No matter what the heat source, thermal efficiency is well under 50%. Stationary power plants have an advantage over portable engines because they can be optimized more for efficiency than weight, size and cost.

Wind turbines will always have high efficiency on paper because they convert mechanical to mechanical (wind to generator shaft) instead of heat to mechanical. In this case, the Earth's atmosphere is the heat engine, converting solar heat to wind mechanical. I think it's more fair to compare thermal power plant EV to the ICE.

Euan, I think you are seriously overestimating the efficency of small ICE. Forty percent is about what industrial scale diesel engines are capable of. The efficiency drops off for the orders of magnitude smaller units used in personal cars. Worse, we have pandered to the desires of consumers for acceleration, and the ability to climb hills at full speed. This means that ICE vehicles are inpractice severely overpowered. ICE achieve the best efficiency at close to full load, but at the power output needed to maintain level road cruising the efficiency is very much lower still. So the efficiency improvement by converting ICE to electric is more like 3-5 times. The real issue, as several other comments have exposed is the combination of battery weight and cost.

Enemy, happy to concede that some of the efficiency factors are out. To do this properly would require more rigorous analysis using a range of efficiencies for different vehicle weights etc. But it seems that the overall picture is broadly correct - though some dispute the validity of using ERoEI as a measure of the efficiency of energy production.

The argument can be made that all journies made by car are worthless and that the efficiency should be zero in each case, in which case it makes no difference which choices we make in future car design.

The post is intended to inform opinion among the media, politicians and the public at large - in addition to the already well-informed opinion of TOD readership.

I pose a few questions in the post - I don't see that anyone has tried to answer these yet.

It is therefore extremely pertinent to ask why EU and US governments have and are still supporting ethanol production for vehicular transportation?

Because they don't want to admit they were wrong, bureacracies take a long time to turn around, and many businesses are now dependent on an ethanol policy. Not that any of that is right or fair, mind you. Notice how politicians turn the crack when the subject comes up now. At the hinner end, they'll have to back off, but probably will not without pressure; the question is, pressure from who? Until we band together loud enough to get some of them munelicht flittin, the ethanol lobby will keep them stroked.

I also agree that 40% is much too high for typical ICE driving efficiency (marine diesels operating at steady peak efficiency can reach this). Also, it is my understanding that oil refining losses are in the neighborhood of 20% and battery charge/discharge losses closer to 10%.

Is the overall picture really correct? Is gasoline ICE really more efficient than hydrogen fuel cell? Both efficiencies are overestimated, but by how much?

Euan,

I have said this before, I think Efficacy is a more correct term, because efficiency really should be reserved for energy conversion systems where in/out can be measured in scientific terms and coherent units.

Lighting engineers refer to efficacy, because what counts is the lumens per watt, similarly cars should be quoted in joules/mile (or km). As you say all the energy is consumed and lost, so the efficiency is zero for both lighting and transport. Its not a critisim, I am just trying to illustrate the point efficacy is a measure of effective "use" of energy but not energy conversion.

Enemy, you may mean this, but to clarify;

I think ICE's achieve maximum efficiency at maximum BMEP. This occurs for a petrol when your foot is flat to the floor and the engine is operating at its speed of maximum torque. This is why higher gearing has improved the economy of cars so much. Oversizing the engine is just another act of consumer demand and stupidity, known as market forces. Diesels are not throttled, so they operate at higher part load BMEP, which is one of several reasons they have good part load economy when compared to petrol engines. Hybrids address this problem by having a small petrol engine operating at high torque, but the price is complexity.

Cars of the seventies had very low gearing, 15-18mph/1000 rpm was typical, the engines just raced away, 30mph ish/1000 rpm is now common but 25-27/mph/1000 rpm has been around since 5 speed gearboxes became popular in the early eighties. This change in gearing has contributed more to fuel economy than any amount of electronic engine management. Try driving a modern car in third gear and you have close a 1970's top ratio, a lot of noise and a big fuel bill!

There is a financial dimension to all this too. The carnage of the last few months has demonstrated the lack of resilience in our capital markets and once again how prone human beings are to Ponzi schemes, even when endorsed or encouraged by government. The existing fleet will only be replaced if there is both sufficient excess energy and finance. Up until now finance has been a very poor proxy for energy, and I expect it to become much more closely aligned along EROI lines in the future. It is not clear at all how this will transpire, but electric cars as capital assets will be MUCH more expensive.

This could be a decades long adjustment. Cuba has provided an excellent glimpse of how things may work out. They have nursed their car fleet well and many 1950's cars are still running. Oil will be around for a very long time albeit less and very expensive. Fewer older gasoline cars with lots of passengers is probably the main part of the foreseeable future.

I agree with you. The financial dimension is what most folk can't grasp. And that is why the ICE will hang on 'till death

Euan Mearns -

While I cannot comment on the estimated fuel cell efficiencies, two things strike me as a bit off regarding the estimates for the wind/electric motor case and the ICE case.

1) An electric motor efficiency of 0.92 is in the right ball park for a very large motor running at its design speed under steady load conditions. but is probably way too high for a small motor operating under highly varying loads from dead stop to full speed.

2) An efficiency of 0.4 for an ICE might be correct for a large diesel operated under steady load, such as in the case of a large marine diesel (some of the really huge ones can do even better), but is probably off by roughly a factor of two for a small automotive ICE operated under widely varying conditions of speed and load, such as would be encountered in everyday driving. I think a more realistic ICE efficiency for everyday driving would be 0.2 to 0.25, at best.

Comment: I'm not sure it's quite valid to set the boundary of the analysis at the end of the engine. The drive train of an automobile downstream of the engine typically loses on the order of 15% of the power coming from the engine output shaft. As electric motors typically do not need to employ a transmission but can be linked directly to the axles, their downstream losses should be less than a comparable ICE set-up. Thus, the boundary of the analysis should really be at the wheels, where the rubber meets the road. I also think it would be more meaningful to show these overall efficiencies as a band rather than as a single number, as there are a lot of inherent inaccuracies in doing this sort of analysis.

Maybe I'm just too dense. Why, Why, Why is everybody still fixated on cars?
I can maybe understand trucks for local transport of cargo once they are offloaded from the trains.

My apologies for doctoring this image from biketrain.blogspot.com/2007/11/help-get-bikes...

Photobucket

Increasing energy efficiency of transport is a necessary but insufficient response to peak oil. We need new models of transport to reduce energy use. Individual ownership of automobiles is likely to become rare. A pool of very efficient automobiles shared amongst many users may be more sustainable. This, in combination with a fixed guideway transit system (rail, PRT, whatever is most workable and efficient), bicycles, and walkable communities may be sustainable.

Hi Euan

Thanks for this "stand-alone" analysis - it is very useful information.

I would agree with several previous posters, however, that your 34% efficiency for the gasoline ICE seems 5-10% too high for an automotive-sized power plants. 76% for an all-electric car seems unachievable too - if energy is being moved hundreds and thousands of miles, transmission losses are bound to go higher than 10%. Incidentally, I think you mean grid transmission EFFICIENCY is 90%, not LOSSES!

The next logical step is to factor overall weight of the propulsion system - i.e. on-vehicle energy storage plus motor(s) and drivetrain plus whatever safety provisions are needed. On this basis, systems with higher reliance on externally-charged batteries still suffer a distinct disadvantage vis a vis weight. And compressed-hydrogen fueled vehicles need a mother of a pressure tank! Safety requirements could turn a nimble H2-powered prototype into a three-ton armored compressed-gas carrier.

Hey guys, be careful lest that pic of the chick with the Fiat interferes with the logical function of our brain cells...

Think I'd feel safer on my motorbike.

I don't see any reason why.

Vision? Getting to the front of traffic?

Big cars and trucks won't disappear anytime soon from Australian roads. People in cars "feel" safer, while motorbike riders (sensible ones) look for danger a thousand times a minute.

How's a tiny, crowded box on wheels going to fair any better in a side impact than me with my helmet, who's more likely to see it coming - indeed, probably wouldn't be there in the first place?

Getting to the front of traffic is more of a whim than safety-related.

You would certainly do better in minor collisions, front and side, in such a car rather than getting your leg crushed or going over the handlebars and risking a broken back. YMMV.

The biggest efficiency loss is the car itself - not the engine. You can build a lightweight car which consumes 1 l per 100 km (about 225 miles per gallon). For efficiency reasons you should not have to bother whether such a car is powered by an electric, hybrid or ICE engine.
Bikes with small efficient motors can consume even less than that.
As long as oil or gas is being burnt to produce electricity that is transported via power lines to charge a battery - electric engines have no real advantage. However if (or when) we stop burning oil to produce electricity, driving electric cars makes a lot of sense.
The easiest way to consume less oil is just to buy fewer cars and to drive less. Although it seems that the industry and politicians do not like that reaction to higher fuel prices.

It is basic understanding in EU a prerequisite of electric vehicles is generation of electrical energy from mostly other than fossile fuels.
This is simply based on total of CO2 emissions.
In the present state of electric power generation a switchover from combustion engines to electric vehicles would at best not change output of CO2.
There is more attention on pushing truck cargo back to rail backed by "intelligent" routing systems
and to ships, too. EU has a high density
of water channels suitable for cargo ships
with 1,000 tons dry cargo capacity.

As long as oil or gas is being burnt to produce electricity that is transported via power lines to charge a battery - electric engines have no real advantage.

Oil isn't used for producing electricity unless you live in some backwards place like Saudi Arabia or Alaska.

Natural gas is easier to distribute by wire as electricity, by truck/train/ship as N-fertilizer and other derivatives if gas is found in a remote area, than it is to build vast networks of pipes to service refilling stations and homes with natural gas. Natural gas is at least as problematic as electric vehicles; energy density is poor unless you liquify it(approximately the same temperature as liquid nitrogen) for vehicle use, which poses it's own problems.

Natural gas is too useful in peaker plants and backup generators to waste it on cars, especially if you plan to have a lot of unreliable generators like wind plants that would otherwise produce near useless power.

Gas-fired generation is a very small part of baseload generation, charge off-peak or opportunistically and the load can be levelized a little bit allowing the share of natural gas on the grid to be diminished.

That episode of 'Top Gear' was just shown on Australian TV. I'd say if the Tesla people wanted good publicity it backfired. The test drive of the Honda FCX in California was more flattering but they omitted some of the uncomfortable facts re price and refuelling while alluding to net energy. To his credit Jay Leno did not seem to go along with the idea that it was a breakthrough design.

An example of hydrogenated synfuel. We can start with water gas C + H2O = CO + H2. If extra hydrogen is added that can be catalytically converted to methane and water (CO + H2) + 2H2 = H2O + CH4 giving easily separated products. Enthalpies are given in standard texts. The extra hydrogen can come from a combination of electrolysis and high temperature dissociation. The combined real world net energy may be lousy even negative, but it will run ambulances, buses, hybrids and some late model ICE vehicles on the road now. No need for radically new or expensive engines or to abandon a huge infrastructure of repair garages and fuelling stations. That's if society still functions well enough to have ambulances and buses when there is no oil.

Maybe besides the point, but now that I've been living without a car for five years (entirely voluntarily), I often wonder why I needed a car before.

I use a community car-sharing service perhaps 6 times a year to transport heavy stuff (or help out friends who never bothered to get a drivers-license).

In the city, a car still takes up a lot of space for parking, garages, service facilities, causes traffic congestion, is a hazard to pedestrians and cyclists, uses scarce resources, even if it's the most efficient wind-powered all-electric sexy vehicle.

Apart from that, it's all policy. If projects like Better Place get sufficient support, and turn out to be successful, and climate change kicks in to boot, I don't see why selling liquid fuel cars will still be legal (or at least cheaper than all-electric) in 10-15 years.

My guess is that all-electric will win hands down, and replace liquid fuel cars to a large extent.

All very well and good, but only if you assume all of our electricity will be generated by renewables. That's a long way off.

Until then, the actual efficiency of an electric car, once you factor in the generation efficiency of a coal or gas plant, is not radically better than an ICE. The difference doesn't seem to be enough to justify the massive changes to infrastructure required.

We're better off in the medium term trying to increase automobile efficiency (by reducing weight as well as by technology) and reducing miles driven, by increasing urban mass transit and putting freight on rail.

Meanwhile we need to increase the percentage of electricity generated by renewables, but let's not pretend that will be a speedy process.

Using wind with ERoEI of 20 gives an efficiency of 95%. Using nuclear electricity with ERoEI of 5 gives efficiency of 80% - so this doesn't make a huge difference to overall calculation. It just seems to make much more sense to use renewable wind for this purpose.

Just for the record, I think the 5:1 EROI for nuclear is based on very biased numbers.  For instance, a nuclear plant takes less concrete per kW of capacity than a wind farm, and the energy in the concrete is repaid in a few tens of hours of full-power operation.  The claims for the mining end are equally suspect; if uranium mining took as much energy as Storm and Smith claim, more energy would have gone to the mines than was actually used by the entire region where they are sited.  And so forth.

Agree, that 5:1 number for nuclear seems to be recycled without much critical evaluation, 20:1 seems more reasonable, even higher for CANDU designs.

Since there isn't enough lead, nickel, cobalt, and possibly politically minable lithium (I think Bolivia has enough lithium, but perhaps not enough government) to make 100 million electric cars, we will have to make methanol for ICE cars, or heat large tanks of high heat containing materials for steam cars.
We won't run out of carbon dioxide, water, or wind/solar/nuke power.
We will run out of cheap oil, gas, and coal.
What balance of electric, steam, and ICE cars we make will depend on the constantly changing technological advances we are making over the period of time concerned. If my geophysical prospecting technology actually works we might make quite a few electric cars. If it doesn't we will be building just a whole lot of methanol plants.
Either way we will be building another whole lot of reactors, windmills, and solar power plants.
We only know that we won't be running 100 million cars on oil in twenty years. The rest will have to be electric, steam, or methanol.

The most efficient cars are actually those cars already made.
We might create the most efficient cars tomorrow, but until we drive the cars that we have today until they cannot be repaired any more. It is actually more fuel efficient to continue using them.

In the short term that must be true.

How many barrels of oil equivalent does it take to make a car though? There must be certain vehicles where the payback mileage for replacement with a more efficient model can't be all that far.

New cars are also safer and pollute less, which is also a consideration.

Hmmmm...replace the whole (doubled) car fleet with electric? Can someone tell me how big the piles of high-toxic, un-recyclable waste we get for replacing every few years the battery? Well, maybe I missed some development and batteries are all recyclable and clean in the meanwhile. What about finite materials for batteries?

Why do we look for a-solution-fits-it-all?

I see the future full of diversity. No matter what scenario (annihilation excluded) - from full collapse to sustainable societies - people will survive and adapt (jeff's rhizome pops up). Cities might be more suited for electrical modes of transport. Long range transport might use different source (hydrogen? compressed air? algaes? whatever, but certainly question mark here), and rural areas might well use local fuels (biofuels can be a solution if in small scale, sustainable and for local needs, it's agri-fuels which are a disaster), and so on.

Finally, efficiency is not always what I look for. If you have a cheap solution, adapted and suited to my NEEDS, for which I am self-reliant and which I can maintain and fix myself, with a terrible efficiency - so what?

What has always held back the electric car is that even for a relatively limited range like 40 miles the batteries required are very large and cumbersome since electrons inherently repel each other, and also have a limited shelf-life and prolonged charging times. The key advantage of the ICE has been that these problems are avoided because energy is stored in the form of the chemical bonds as gasoline/petrol in a relatively light easily stored liquid. I think an efficiency % plot is an overly simplistic approach to weighing up the relative merits of each approach. With an abundant supply of energy, probably from nuclear fission and eventually fusion in addition to renewables, efficiency % would not necessarily be the primary concern and the hydrogen fuel cell might still win out in the end if the various engineering issues can be overcome.

The Planté cell is ancient, true, but not even it has escaped the recent trend of radical improvements in batteries.  Firefly Energy has commercialized a vitreous carbon foam backing to replace lead grids, allowing all the non-active lead to be dispensed with and slashing the weight of a cell by more than half.  This also increases the cell life; the carbon backing cannot corrode and fail like lead grids, and the small cell size of the foam limits the size of sulfate crystals.  Power capacity is also increased.  The batteries can be recycled like standard lead-acid units (the carbon burns off in the molten metal bath).

I keep wondering why Firefly hasn't had anyone decide to license their technology for a cheap killer PHEV.

The future of motor vehicles lies in improved efficiency and that is to the left of the gasoline ICE in the chart. That future is electric vehicles powered by high ERoEI renewable electricity.

I think the idea that we could have an electric economy powered by renewables is a bit unreasonable.

http://geocities.com/rethin/

Many factors are outside the range of this small model. Oil production may decline at a much more rapid rate than modeled. Coal reserves might be more limited or coal fungibility much smaller than expected. Perhaps nuclear will undergo a renaissance and make a significant contribution to energy demand. It is possible that a future electric economy will be so efficient that no per capita energy growth will be needed at all (although even this scenario demands 8,738 MTOE from wind/solar in 2050).

The point of this hyperthetical model is to see what demands continuing our present happy motoring lifestyles will place on wind/solar in the future. If the world continues along its present development path it seems the demands we will be making of wind/solar will be very very large and historically unprecedented. The historic growth of any source of energy, singularly or combined, doesn't even come close to the growth rates we will need. For example, even with the conservative growth scenario 2.0, we'll demand wind/solar to grow nearly three times as high and in one half the time as oil historically has.

Additionally the historic growth of any one energy source has always been supplemental to the energy sources that preceded it. The demands on wind/solar will come during a period where all other sources of energy in aggregate are declining at a significant rate. Additional hurdles remain in transitioning our transportation system, economy, and society off of liquid fuels and onto a new electricity driven form.

Perhaps its time we put a rest to the notation that continued world energy growth can come on the backs of wind and solar. To steal a simile, if building our fossil fuel economy is like putting a man on the moon, building a post fossil fuel electric economy will be like putting a colony on Pluto.

Has anybody seen wood consumption estimates? When we had the oil embargoes of the early 70's and 80's, there were many people converting to wood stoves for heating.

Recall the great post by Nate Hagens about wood in the USA and how long it would last if it was all we had to use for heating:

Home Heating in the USA: A Comparison of Forests with Fossil Fuels
http://www.theoildrum.com/node/3374

See the concluding graph "Number of years of heat from forest alone (no fossil fuels)"
http://www.theoildrum.com/files/yearsofheatdata2.JPG

-- Philip B. / Washington, DC

That future is electric vehicles powered by high ERoEI renewable electricity.

I think I could maybe have phrased this better. What I was trying to say is that ethanol and H fuel cells do not have a future in cars. If cars do have a future then it will be electric. And having electric cars in future does not mean we will have more cars.

I agree with what you say about the challenge of building renewable infrastructure. But if we make the wrong choices now then we have no chance of a future.

But if we make the wrong choices now then Since we made the wrong choices 30 years ago we have no chance of a future.

Even no growth of energy per capita requires a pretty much impossible growth curve from renewables.

Couple this with the large upfront costs in money and energy that wind/solar require and their slow payback rate I think even our current standards of living are impossible.

There is just no future in cars, electric or otherwise.

Even no growth of energy per capita requires a pretty much impossible growth curve from renewables.

If you cheque out Luis' Olduvai 2008 revisited you'll see that we envisaged 50% energy savings via efficiencies. Its quite clear that current levels of energy consumption are not going to be viable.

http://europe.theoildrum.com/node/3565

The payback rate for high ERoEI sources is very rapid.

If you cheque out Luis' Olduvai 2008 revisited you'll see that we envisaged 50% energy savings via efficiencies.

That might be true for the west. For instance the US uses 7.9 TOE per year. But China uses 1.24 and India only 0.53 toe per person.

All those billions of IndoChinese are going to be demanding tvs, air conditioning and even cars/suvs. There is no way you are going to see a decrease in their energy usages. Even a slight per capita increase in China/India is going to way way offset a dramatic decrease in the West's usage.

Check out the three C's in Japan, air conditioner, color TV and car.
http://books.google.com/books?id=BvUEzBin61AC&pg=PA334&lpg=PA334&dq=japa...

Moving from a bicycle to a car, even if its an ultra efficient electric car, is going to increase your energy usage, efficiency or no efficiency.

Also, no matter how rapid the pay back rate for wind/solar it is still much slower than with traditional fossil fuels. While you are doing a dramatic ramp up you are always going to be behind the curve and will only catch up when the build up slows. By that I mean you are using the current generation of wind/solar to build the next generation. Its only when there is no next generation that the current and past generations of wind/solar are freed up for the rest of the economy.

Rethin,

"I think the idea that we could have an electric economy powered by renewables is a bit unreasonable."

You are making a common mistake, thinking that renewable energy will have to replace the energy content of oil and coal. Coal is burnt to generate electricity, at 35-50% efficiency so renewables only have to replace the electric energy burning this FF would generate.
Euan has demonstrated that replacing oil for transport by EV's would only require about half as much energy.

To replace most oil used in land transport, with electricity would require a 20% increase in todays electricity production. If that's done over 20 years, would require to increase electricity production by 2-3% a year or increase efficiency of lights and appliances and insulation by 2-3%. Both seem possible.
Considering that less than 10% of the worlds hydro capacity is being used, wind and solar are growing at 30% a year, and nuclear energy is growing, its not wildly optimistic to think that non FF can replace the "work done by oil", if not the energy content.
Replacing coal generated electricity is going to be a bigger challenge, but will probably have more than 20 years to do this.

Its also possible to save at least half the energy currently used to heat buildings, and make use of all the waste heat which goes up cooling towers using smaller CHP power plants.

You are making a common mistake, thinking that renewable energy will have to replace the energy content of oil and coal. Coal is burnt to generate electricity, at 35-50% efficiency so renewables only have to replace the electric energy burning this FF would generate.

The numbers are all normalized thanks to the BP statistical review. I don't remember what the conversion efficiency was off the top of my head, but its in BP's spreadsheet.

So a TOE of coal is equivalent to a TOE of oil which is equivalent to a TOE of electricity.

Now you can argue that BP doesn't have the right conversion factor, and some have done so. But you really can't accuse me of missing it entirely.

Rethin,
Looking at the link you provided, today about 6,000MBOE energy is used to generate electricity, and by 2050 the projection is for another 11,000 MBOE needed for growth and replacing most oil and gas. Thus today's electricity production( approx 2,000GWa) needs to increase to 5500GWa or increase by 90 GWa a year, to give the expected 50% growth in energy use(MBOE basis), allowing for declines in oil and NG.

Now considering that China has 100GW hydro capacity under construction, 11GW of nuclear under construction, with 27GW planned and 58 GW proposed its not a big stretch that China could manage 22GWa new hydro and nuclear per year, (25%required world growth). Last year, 28GWc(9GWa) of wind was built. Is it really a big stretch to expect wind and solar capacity additions to increase x6 fold in the next 20 years? or even in 10years?

The paper I wrote says

How much energy will the world require in 2050? To answer this question I looked for world per capita energy data. This site shows the IEA World Energy outlook 2006. The world is currently at 1.69 TOE (ton of oil equivalent) per capita per year. For comparison the US has a per capita energy usage of 7.9 TOE per year, Europe clocks in at 3.8 and the Japan is currently at 4.17 TOE per person per year. The IEA predicts world per capita energy usage at 2.1 TOE/person/year in 2030. I decided to be conservative in my experiment and used a value of 2.0 for 2050. A per capita energy usage of 2.0 in 2050 is consistent with continued growth in total energy of 1.2% a year. Obviously a higher per capita figure would put even more demand on wind/solar.

With world population projected at nearly 9 billion in 2050, a per capita energy usage of 2.0 TOE would require nearly 18,000 MTOE. Unfortunately non-wind/solar energy in our baseline model will have declined to 6,500 MTOE in 2050. The difference, 11,500 MTOE will have to be made up from wind/solar.

I'm not really sure what it is you are referencing.

But if you are asking me if I think renewables can make up for the decline of fossil fuels then no, I don't think so at all.

So wind/solar will have to go from 40 MTOE from 2006 to 11,500 MTOE per year in 2050. I'm not sure where you are getting 6x from, but its not from my paper as far as I can tell.

Rethin,
Going back to the data you have used for the graph, I have in 2008 nuclear (700MTOE,) Hydo (700MTOE),Oil(3700MTOE),Coal(3400MTOE),NG(2600). If we add up nuclear, hydro, most coal and one third NG we have 5,450MTOE, the energy used to produce the 18,500TWh(2,100GWa) of electricity generated according to CIA estimate for 2008.
To increase non FF energy by 11,500MTOE(your figure) we would have to generate an additional 2x18,500 TWh above what is generated today. This works out at 4,200GWa additional power, or adding 100GWa new production per year for next 42 years.
In the last 20 years nuclear has been growing at an average of 10GWa a year, hydro at least 10GWa per year. If the rest is going to come from solar and wind, that's 80GWa per year. Last year the world added 9GWa of wind, but it's growing at 30% a year so in 10years could easily be adding 90GWa wind( and 10GWa solar).
So by 2050, would have an additional 400GW nuclear, 400GW hydro(less than 10% of undeveloped capacity), 3100GW wind and 340GW solar.
I believe that hydro, nuclear and solar will probably grow quicker than this, since China alone seems to be developing 5-10GWnuclear and 5-10GW hydro per year, and world wide,solar is growing at 50% per year.
Is it credible that the world can have 3100GWa of wind power by 2050? Manufacturing 90GWa per year would require to build 270,000MW capacity per year, at 100 tonnes of steel per MW, or 27Million tonnes. This is 2% of the worlds annual steel production. Estimates of world wind resources, where the wind is >7m/sec, are >72,000GWa, so 3,100GWa would be about 4%.

To increase non FF energy by 11,500MTOE(your figure) we would have to generate an additional 2x18,500 TWh above what is generated today.

I wrote:

With world population projected at nearly 9 billion in 2050, a per capita energy usage of 2.0 TOE would require nearly 18,000 MTOE. Unfortunately non-wind/solar energy in our baseline model will have declined to 6,500 MTOE in 2050. The difference, 11,500 MTOE will have to be made up from wind/solar.

We would need 18,000 MTOE of energy in 2050, but FF and nukes and hydro sources will only be able to supply 6,500 MTOE in 2050. The difference, 11,500 MTOE will be needed from wind/solar in 2050.

You can see in the graph the blue and purple are from nukes and hydro, they are taken into account already. You can read GuiderGlider's excellent work linked to from my paper to see he comes up with these numbers.

It would help if you took the time to read both papers to get a better understanding of the work that was already done.

Rethin,
I have read the paper by GuiderGlider, which is based on a future scenario of no more nuclear energy than today, a modest 40% increase in hydro and what appears to be a 20% per year growth projection of wind and solar.
Converting GG's figures from nuclear MTOE to GWatts(capacity) based on 2008 figures( that are published in GWh or GWc) it seems that GG is projecting: 1)Nuclear to go from 372GWc to 472GWc by 2025 then decline as older plants are shut down. China and India have more nuclear capacity under construction than they have operating and have plans/orders for an additional 300%. Japan, S Korea and Russia have 25% additional capacity under construction. I see no credible reason why China and India will not generate at least 20% of their electricity by nuclear in 2025, and continue to expand beyond 2025 if LNG becomes expensive or unavailable.
If the EU starts to have NG shortages by 2025 as GG predicts is it not likely that existing plants will be kept going or replaced? I would predict that the EU will start building again after a few winters with rolling blackouts, until nuclear accounts for 50% of electricity consumption. Alternatively, everyone will move to France for winter holidays.
2) Hydro, GG is projecting today's 600GWc to increase to 824GWc,in 42 years, even though China has 100GWc under construction which will give a 70% increase, and less than 10% of world capacity has been developed. Alaska for example has only developed 1% of its 45GWc ( excluding federal lands). When the oil and NG is gone surely a significant part of that will be developed in Alaska. Similarly for Asia, S America, Africa and Canada. A 40%increase in hydro per decade would seem more reasonable except in EU.
3)Wind, GG mentions the 30% pa growth rate for wind but uses projections based on world industry energy association projections using 20% growth rate that is already( 15 months later) out of date, projecting 109 GWc for 2008 when in fact it is 121GWc(29%increase). Thus the projections for 2050 of 1,000MTOE at 20% growth would be reached by 2022 at 30% growth.
While nobody can predict what future growth rate wind will have, you have compared the growth needed for wind and state this would be historically unprecedented. In fact if you go back to GG's graph of nuclear energy from 1965 to 1988, nuclear power also doubled every 3 years and only slower after the accident in the FSU, but still increased 50% in next 20years.

Your graph(fig 3) of MTOE wind per year indicates that after 2026 "growth" is steady, increasing at 300MTOE per year(200GWa)until 2050. If this is what you mean to indicate, that would represent a yearly addition of 600GW capacity. At 30% growth today's wind 27GWc addition will grow to 600GWc addition per year by 2019. If this capacity addition is reached in 2019, 2026 or even 2031 the message is a lot of oil and NG energy can be replaced by wind(and also hydro and nuclear). This is assuming BAU, if oil declines faster, we may see much faster build out of nuclear and renewables and perhaps even an increase in gasoline taxes or rationing!

While nobody can predict what future growth rate wind will have, you have compared the growth needed for wind and state this would be historically unprecedented. In fact if you go back to GG's graph of nuclear energy from 1965 to 1988, nuclear power also doubled every 3 years and only slower after the accident in the FSU, but still increased 50% in next 20years.

This is the graph showing historical nuke growth vs wind/solar.

Nukes barely get off the x axis. You can see its hardly the sort of stellar growth we'd need from wind/solar.

Its not much to claim that an new industry shows large exponential growth when it is still small. That's just because the existing resource base is small, so any growth looks large compared to it.

There are all sorts of natural limitations on why large exponential growth is nearly impossible to sustain as an industry matures. I've had this same argument with others who claim there will be no practical limits on energy growth in the future. I'm not sure I really want to do it again with you at this time. But to give one example there is only one company in the world that still has the expertise to build the reactor vessels for nukes. Its a small steel company in Japan and is only capable of turning out 4 such vessels a year. That right there is just one credible reason China/India won't be generating 20% of their elec a year in 2025.
http://www.bloomberg.com/apps/news?pid=20601109&refer=home&sid=aaVMzCTMz3ms

Rethin,
I am not critical of your approach, just some of the assumptions you use, for example, India only builds CANDU type(heavy water) designs, that don't have a large pressure vessel. All are built in India. China also has some CANDU reactors and a Russian reactor.
Russia is planing to double nuclear power by 2020, using WER design. As far as I know, it doesn't use Japanese pressure vessels for its WER reactors, at least not the ones being built in Iran.
The link you gave mentions that the Japanese company is back-ordered to 2015 and plans to double capacity. That should tell you the industry is expanding rapidly. Is there a reason why the company cannot quadruple capacity in 5 or 10 years or produce X10 capacity in 20 years?. Perhaps another company will start building pressure vessels! We are talking about a 40 year period. The US built 100GW nuclear capacity in 20 years, under conditions of a shortage of enriched fuel. This was a limitation to PWR expansion in 1970's until the perfection of gas centrifuge enrichment.
Wind energy growth at 30% per year will also continue to have new capacity constrains, and like the nuclear industry each one will be solved if the demand continues to grow. The big advantage of wind and solar over nuclear and hydro is the rapid construction time.

Drop me an email in 2050 and I'll be happy to concede.

Citeth Rethin:

That graph is utterly bogus.  Nuclear produces far more electricity than wind and solar put together, and if non-electric power is being rolled in, it's an apples-and-watermelons comparison.

Try reading the paper I pulled it from.

I did so.  You failed to reference your sources in a way making it easy to distinguish where your numbers came from (and you changed the X axis along the way); I finally found the specifics here.  Once I had the data I found that you were using projections out to the year 2050.  The projection for nuclear is that it will be producing less power in 2050 than it does today!  Given the recent conversion of Lovelock and other ecologists to nuclear advocacy, the imperative to decrease GHG emissions, and the massive building programs of both China and India, I find this ridiculous.

The USA gets roughly 90 GW average from nuclear, but has enough fissionables in spent LWR fuel alone to start up perhaps twice our current capacity using IFRs and LFTRs.  Once started, these reactors would not need enriched uranium, or any newly-mined uranium at all for hundreds of years; spent LWR fuel, DU from enrichment and thorium would be their only required input.  We should be looking for nuclear to have eclipsed both oil and coal and rival wind (which is actually synergistic with nuclear) by 2050.

Then you completely failed to see the point of the paper. It was an extension of the conversation that followed GG's post looking specifically at only wind/solar using GG's model as a baseline.

The above graph is the historic growth of nukes 1965 to 2006. There is nothing bogus about it at all.

All of this is spelled out very clearly in the paper which you obviously hadn't bothered to read before you started to call things bogus.

And all the data sources are clearly linked.

Rethin, Don't take it to heart, just play them at their own game.

Here's a quote from Paul Simon from his song "the boxer"

"A man hears what he wants to hear and dis-regards the rest" you could replace "hears" with "reads"

Its a major problem here that folk will try and make you look like a fool, even if your point may be valid. R Waffle above is a classic example, I quote 800 watt hours for 55 mpg, he is up in arms because the true figure is 900 watt hours. The fact that diesel contains over 330 times more energy than a battery, he conveniently dis-regards and misses the point I was making completely.

Its very annoying, frustrating, but one has to accept we are dealing with folk. When your decorating your house like my missus is forcing me to do at the moment, drawing people into a good argument can be therapeutic.

Good luck in putting your case forwards!

No, what you're showing is mostly projections of growth for decades beyond the present based on assumptions you're not even trying to justify here, just presenting as gospel.

Here's the real historical data for wind and nuclear, from the EIA (and you wouldn't believe what a b***h it was to get this input and formatted correctly):

Are you drunk?

Its a simple thought experiment. The purpose was clearly spelled out in the introduction.

The historical data is from the BP statistical review and Romer. The future projections aside from wind/solar are from GG's work. But this too is clearly spelled out in the paper.

Good grief man, you are just inventing things at this point.

But earlier, you said:

Nukes barely get off the x axis. You can see its hardly the sort of stellar growth we'd need from wind/solar.

If you didn't mean that, you shouldn't have said it.

Nukes have shown exactly the kind of rapid growth curve we need, and have continued to increase net output even as plants are decommissioned.  Wind is about where nuclear was in 1970, so it should follow with roughly a 40-year lag.  PV is the wildcard, as it can be installed at point of use and requires no planning permission for transmission lines; it is about 0.01% of US generation, but could explode very quickly.

That's historical data. You can't argue with it.

Nukes historically have not shown anything close to the growth we'd need under the scenarios I presented in my paper.

And as I stated before, with a tiny installed base just about any industry shows large exponential growth early on. But the fact remains that over the last 40 years nukes have shown nothing but anemic growth at best.

If wind shows the same kind of growth we've gotten historically from nukes then we are in for a very severe energy shortfall.

Do you have a coherent point to make? This is getting tedious.

That's historical data. You can't argue with it.

Duh.  It disproves your claim.

Nukes historically have not shown anything close to the growth we'd need under the scenarios I presented in my paper.

You think that the growth from 7.7 billion kWh in 1967 to 806.5 billion kWh in 2007 (a 12%/year compounded growth rate) isn't sufficient?

The growth of nuclear was constrained by the coal industry and its propaganda and paid legal obstruction (lots of anti-nukes were financed by coal interests); once it was done pushing petroleum out of all but niche markets, it had a much tougher competitor.  The political and physical climates have both changed, and coal may not stand in the way of nuclear any more.  If the USA can add 10 GW per year (33 300 MWe modular reactors, for example), that's enough to replace coal in 20 years.  What growth rate do we really need here?

If wind shows the same kind of growth we've gotten historically from nukes then we are in for a very severe energy shortfall.

Growth of any source is either going to look like a logistic curve (exponential early on, trending to linear as it gets to the halfway point, then asymptotic to the limit) or boom-and-bust.  It's the way things are.

You think that the growth from 7.7 billion kWh in 1967 to 806.5 billion kWh in 2007 (a 12%/year compounded growth rate) isn't sufficient?

Yes! Very much this!

Its the conclusion of the entire paper. No source of energy has historically grown anywhere close to the rate we'd need wind/solar if we are going to rely on them in the future to give us anything close to BAU.

And its not just that nukes don't show the kind of performance we'd need, but they are the worst performers of all the energy sources. In 40 years of growth nukes don't get over 600 MTOE a year. Compare that to Oil and Gas or even coal in figure 10. Gas in a 40 year period grows nearly 2000 MTOE a year between 1960 and 2000.

Yet even all fossil fuel combined historically fall far short of the growth needed if we were to rely on wind/solar as our future energy sources. See figure 8.

Did you even read the paper?

In 40 years of growth nukes don't get over 600 MTOE a year.

Your own source says that nukes made more than that (675 mmMTOE) in 2007; it projects 821 mmMTOE in 2021!

Yet even all fossil fuel combined historically fall far short of the growth needed if we were to rely on wind/solar as our future energy sources. See figure 8.

I'm not arguing that wind and solar are our only prospects.  Are you?

Did you even read the paper?

No need; it's based on faulty projections (and based on what you said above you don't even use those correctly).  Garbage in, garbage out.

So you didn't even read it.
I am not wasting any more time with someone who's mind is so obviously closed.
How in the world can you waste so much time and energy arguing about something you didn't even read? Pathetic.

If you'd bother to actually read it a lot of you mis-conceptions could be cleared up very easily.
It always boggles my mind when I come across an obviously intelligent person that insists on arguing a topic yet refuses to read the data at hand.

I don't mind having an honest disagreement. But this is ridiculous.

So you didn't even read it.

You referenced the graph here, not the essay as a whole.  I said the graph (and the data behind it) was bogus, and I have supported this opinion with facts.  You based your essay on this data.  One of the consequences of basing an essay on bogus material is that it is not trustworthy, perhaps "not even wrong"; it's certainly not worth critiquing until it's been corrected.

If you don't like the facts, it's not my fault.

You are insane. The graph was historical data from the BP statistical review 1965-2006.
It is available right here
http://www.bp.com/productlanding.do?categoryId=6929&contentId=7044622

The graph you posted was not historical data, it was projections out to 2050 (and with an X axis that isn't even marked by year).

If you don't even know what you're posting, you are not one to question others' sanity.

(dup deleted)

(dup deleted)

Hey, Rethin, "But if you are asking me if I think renewables can make up for the decline of fossil fuels then no, I don't think so at all", that is a ridiculous thing to say. Because unless you know of some secret energy source, renewables replacing historic fuels along with human adaption is exactly what will happen. There is no other outcome possible. Nuclear fission does not have the expandability, nuclear fusion is a maybe, and geothermal is an assist. Apart from those, all energy comes from the sun on a daily basis.

renewables replacing historic fuels along with human adaption is exactly what will happen. There is no other outcome possible.

There are other possible outcomes. Collapse and die off for instance.

I'm in favour of electric cars, but one thing I've wondered about is how long it will be before the question of how to tax electric cars for road use arises.

Road tax is a significant component of the cost of petrol in several countries. I assume the few electric cars on the road now recharge on domestic power and get a free ride, but surely this is only because they are under the radar. As the number of electric cars increases and road tax from conventional cars falls, there will be pressure for governments to do something about it. It seems inevitable.

The economics of cars like the Volt will also change. At present the assumption is that you'll pay a lot up front but the car will cost very little to run.

Battery powered Vehicle
End performance -------------- 100 reqd input
engine conversion -------------- 90% 111
battery discharging ----------- 85% 131
battery charging -------------- 65% 201
electricity transmission -------- 93% 216
conventional power generation -40% 541

overall efficiency --------------18%

Now, change the assumption to wind power:

Output Efficiency Energy, %
End performance 100
Motor conversion 90% 111%
Battery, Li-ion 95% 117%
Battery charger 90% 130%
LD electric transmission 85% 153%

Overall efficiency 65%.

Curious as to what "efficiency" factor you would put on wind power???

Li-Ion rate may be good for new batteries, but decrease for older ones. (that's what the EV manufacturers don't tell you)
Your Battery charger number is high as well. But there is not the only the charger, but also the charging of the battery: a wide spread depending on discharge levels and desired charge speed, etc.. Not negligeable at all.
You're right on long distance electrical transmission.

Curious as to what "efficiency" factor you would put on wind power???

It makes as much sense to ask that question as it does to ask how efficiently a supernova turns its energy of collapse into available uranium and thorium 4.6 billion years later.

I see, basically a none answer.

If you think it's important, tell me why it matters.

An estimate from some time ago was that the continental USA was good for about 1.2 TW of wind power (about 3.3 times current national average consumption).  That's at the busbar, not blowing by.  What difference does the efficiency of producing that 1.2 TW make?

When I suggested (in a post above) 90% eff. for a battery charger I got slated. It gets very confusing does this electric vehicle technology. I suspect the electric vehicle suffers the same exaggeration of performance as do current cars. So in reality, knock 30% of the manufacturer's claimed fuel performance figures and you probably won't be far off. That makes your 153% closer to 200%

Euan,

Thank-you for an excellent post (as usual). I believe that your conclusions are a wake-up call, but I thought I would mention a minor quibble (doesn't subtract from your overall conclusions). I was recently privileged to spend a year working for a "hydrogen energy" firm called Millennium Cell. (now defunct). They wisely concentrated on the market of portable power from hydrogen fuel cells and did an excellent job with the chemistry of using the hydrogen from sodium borohydride released in aqueous solution "on-demand". It seems that hydrogen HATES to be stored with itself (e.g. compressed or liquified) and LOVES (chemically speaking) to bonded to other "metals" (like B, Al, etc.). However, the economics of hydrogen energy (even with a metal to bond the hydrogen) are just not good as you have already shown. Our experience at MCEL was that the best practical efficiency that one could obtain from a fuel cell as actually only 40% (not 50%). That is due to a range of factors including the need to "flush" the fuel cell periodically to avoid build-up and blockage. It was also very difficult to maintain the exact amount of moisture content for the membrane (where the hydrogen reacts with the oxygen from the air). Cold temperatures were particularly difficult. Any temperature below freezing was literally "death" for the fuel cell (if the membrane freezes it cracks and the hydrogen doesn't then react, it just passes through). I notice that fuel cell vehicles are not often marketed in cold climates. Very hot and dry climates are just as problematic (if the membrane dries out it is just as damaging to the future performance). If you think batteries are a problem in the cold, try operating a fuel cell in Canada in January!! (or at least one that isn't heated all the time)

I just wanted to quibble that the fuel cell efficiency that you mention (24%) is a little optimistic. It is probably more like 19% (or less in colder climates).

Keep up the excellent and informative posts!

Ian

Thanks, I dare say one could quibble with a lot of the numbers, but it seems the overall ranking is right.

I'm not sure how electric cars would fare in a very cold climate either, since staying warm inside may be a problem. They would need to be designed with good insulation.

I dare say one could quibble with a lot of the numbers, but it seems the overall ranking is right.

All quibbling aside, a ranking that has Ethanol at a lower ranking than straight Gasoline must be wrong.

And a battery efficiency of 97% is fantastic, i.e. pure fantasy.

I'm no expert on batteries. I posted a link to my source - if it is fantasy then maybe you could post links to more instructive sources.

In comparing energy efficiency of gasoline to ethanol you need to consider an oil well producing 50,000 bpd versus combusting food in a traffic jam.

Neither of those options are sustainable. But I believe that wind (in windy places) and direct solar (in sunny places) are - so long as the population and consumptive pattern metrics are controlled.

Here is another link for you then: http://xtronics.com/reference/batterap.htm

The link you posted merely considers different modes of "depletion" for a certain type of battery. The author calls that the "efficiency rate". But of course that notion doesn't say anything as to how you get power into your battery. For that you need a battery charger that converts AC into a certain DC voltage, which has certain power factor and efficiency characteristics.
Then also the battery itself during charging doesn't store all the power received from the charger: it looses some. So, the charging process alone has significant power losses. Then the discharging (=utilization) process has some losses where not all the power that was stored enters the actually electric motor.

So, all power that came out of your garage outlet certainly does not come into your the electric motor of your car. And it would be relatively far from 97%.

In comparing energy efficiency of gasoline to ethanol you need to consider an oil well producing 50,000 bpd versus combusting food in a traffic jam.

Oh really? Is that the "logic" you applied to the other alternatives as well? And I thought I was missing something.

Very good paper, Euan. Wait for my post where I'll show my brand new Citroen Ax all electric car (well, not actually new, it is 13 years old, but an electric car, at 13, is still a baby). Powered by the latest generation lithium batteries, top speed 95 km/h, range more than 100 km, what else can I ask? I am fully prepared for the next oil shortage. But, one question, what kind of car do YOU drive?

I drive a 14 year old Volvo 850 estate held together by the smell of dogs.

Its had some problems recently and I was going to replace it. But I took it to a country garage and got it fixed for 25% of the cost quoted by Volvo and its running better now than for a number of years.

Extravagant I know, but a key issue with the life cycle energy budget of cars is the embedded energy. Do you know of work done illustrating this point - life cycle energy costs for manufacture, running and service and disposal. I think making things that last is important.

Ugo, we look forward to the post. 10,000 miles of commuting on your electric scooter has been an inspiration to us.

Some more numbers:

1 horsepower = 750 Watts.

A typical car puts out 100HP peak = 75KW, and around 5-6HP average = 4KW

A typical cycling human can generate 500W peak, 150W average.

Electric-assist bicycles are a possibility for future personal transit, I suspect electric cars are not.

I sometimes feel that explaining power consumption to a Westerner is like explaining swimming underwater to a fish.

As an antidote to Jeremy Clarkson....

Jay Leno test drives his 1909 Baker Electric:

http://www.jaylenosgarage.com/video/video_player.shtml?vid=187711

(pity about the sponsor's ads)

BobE

"ERoEI = energy procured / energy used to procure energy"

I wonder. What is the energy procured in the case of a motor vehicle? Whatever the answer is, it is unlikely to be much related to the social or personal utility of the motor vehicle. ERoEI is useful for different energy resources and the technologies for exploiting them, but for motor vehicles? Really? I wonder.

For energy resources, it is required that the value of ERoEI be greater than one. For motor vehicles, ERoEI must be less than one.

Let's define some more terse variable names and do a little algebra.

ER = energy procured
EI = energy used to procure energy
EF = 'efficiency"

With these definitions

ERoEI = ER/EI

"efficiency = (ERoEI - 1)/ERoEI" becomes

EF = (ER/EI - 1)/(ER/EI) = 1 - EI/ER = 1 - 1/ERoEI

For motor vehicles, ERoEI ranges from 0 to 1, regardless of how ER, 'energy procured' is defined. Surely one wishes the motor of a motor vehicle to produce the most power for the fuel consumed. This translates into wanting ERoEI as near to 1 as possible. But this leads to EF near zero. Is this an intuitively reasonable definition of EF? It gives highest rating to a Yugo with blown head gasket and a leaking petrol tank.

For ERoEI greater than one, this definition of EF is more intuitively reasonable, but still suspect, as it has no basis in the traditional technical meaning of 'efficiency', the word. It is just a transformation that maps the range 1 to infinity onto the range 0 to 1.

I made a mistake. Things are worse than I thought. This definition of EF maps the range of ERoEI from zero to one onto the range of EF from negative infinity to zero.

ERoEI = 1 goes to EF = 0

ERoEI = 3/4 goes to EF = -1/3

ERoEI = 1/2 goes to EF = -1

ERoEI = 1/4 goes to EF = -3

ERoEI = 0 goes to EF = -(infinity)

Dr Hall and EROI Guy have a paper posted that explains how to handle mixing efficiency and EROI.

The idea of making cars more efficient by making more efficient cars is sheer folly. I can take any pick-up truck and increase its fuel efficiency one or two thousand percent just by breaking a few laws. First, you pack about a dozen people into the bed, standing shoulder to shoulder like sardines. Second, you drive about 25 mph, down the highway, because going any faster would waste fuel and wouldn’t be safe with so many people in the back. And there you are, per passenger fuel efficiency increased by a factor of 20 or so. I believe the Mexicans have done extensive research in this area, with excellent results. -- Dmitry Orlov, author of Reinventing Collapse

Jokes aside, here in South Africa the minibus taxi industry provides a good alternative to personal vehicles. The latest taxis introduced with government support are comfortable, have aircraft-style seats, and cost just a little more than the subsidised buses. On a liters per passenger-mile basis, it must be one of the most efficient transport systems ever.

I believe future personal vehicles will be eggshells on wheels, being very light and streamlined to be more fuel-efficient. They will of course be less safe in a collision. It's a price the non-elite will have to pay.

here in South Africa the minibus taxi industry

With the added bonus of helping to reduce the burden of over population at the same time ! Joburg teksis are the 9th wonder of the world for those that haven't sampled the pleasure of being run off the road by one doing a u turn trying to pick up that vital 40th passenger on the motorway.

http://www.taxi2000.com/

No battery.
Vehicles weigh only 400kg.
Journey is non stop.
Only 16 moving parts in the thing.

There used to be more info on the site.

I can just see broken governments and insolvent banks lining up to pay to build those systems!

we need to go to Alan From Big Easy's electrification of rail ideas - shovel-ready projects that were built with shovels and sweat and horses the first time - so we can certainly redo it now (while we still have convenient energy sources).

I don't see any "TO THE FUTURE AND BEYOND!!!" systems getting built while GM goes under, Ireland defaults etc. etc. etc.

but an electrified train and light rail system combined with bicycles and shoe leather could keep people moving to jobs, and food moving to people for a very long time

My idea to increase the efficiency of electric cars is to mount pedal-chargers at the front and rear passengers seat. The passengers could pedal-charge the batteries as they were transported. Kills the obesity and energy efficiency problems with one stone.

My idea would be to electrify the existing vehicles ...

If you wanted a large suv , you would have to bay the expense

smaller cheaper cars like vw or geo would be best

http://www.zoomilife.com/2009/02/08/michigan-man-creates-electric-volksw...

bicycle .... even better

http://www.zoomilife.com/2009/02/16/drymers-innovative-three-wheeled-ele...

55 mph max speed limit

Most electricity is powered by coal. If one were to speculate back in 1890, one would be looking at an ERoEI of a coal power plant generating electricity of somewhere in the range of 35% - 40% (considering line loss of electricity). Then, using that info one might suggest that no coal powered electricity plants be built. Considering everything, a significant contribution from hydrogen should not be disregarded. If hydrogen is produced from wind, solar, wave and nuclear, then, despite the low ERoEI, it is probably a strong possibility in the face of no more low-cost oil available.

All this "electric car" speculation forgets how the real world works. Out there it gets to -40C, or colder. Maybe for a month. Maybe more. Batteries are worthless at these temperatures after they get cold soaked.

More to the point, the car requires HEAT. Lots & lots of HEAT. 10kw would hardly warm your tootsies...

Does one have an ICE to keep warm? And don't forget the windows need heat too...

Lots of heat. From a battery? Or from the tooth fairy...

If you visit Winnipeg, Canada, which often has -35 sometimes -40C,in winter, you will be amazed to find several hundred thousand cars all with 12V lead acid batteries. They often have electric frost shields on back and side windows to keep inner glass free of ice. My vehicle didn't get warm enough to thaw out ice on exterior all winter because I parked outside( plugged in electric block heater to keep oil from freezing). Fully discharged batteries can freeze over several days, so can diesel fuel.

Regarding sources of auxiliary heat in electric vehicles, two come to mind immediately. First, the 15% "efficiency losses" of the drivetrain are almost entirely dissipated as heat in the motor and drive controller, which should yield perhaps 15 KW x 15% = 2.25 KW of cabin heat, surely enough to suffice if well insulated. Second, the "next big thing" in auto air conditioning is CO2 compressor circuits. Turns out that with a CO2 circuit rather than a freon circuit, air exchange heat pumps are very usefully efficient right down to northern winter cold temperatures. even with a COP of only say 2.5, it could provide cabin heat of 2.5 kw drawing only 1 kw from the battery, or perhaps only 7.5% of the drivetrain power at highway speeds.

Snow Bird you are correct. The rating for decent a car heater is at least 10 kW, and at idle even an ICE (particularly diesel) struggles to supply them with enough heat for maximum output and that's at UK winter temperatures.
Those who want to insulate cars should wear water proofs. Most car heaters draw in cold air from ouside, otherwise the car steams up due to condensation. This is the problem when reciculation mode is used.

A pesimist is an optimist who is aware of the facts!

The trivial solution to humidity is to run the A/C but dump the heat back to the passenger compartment.  Voila, dehumidifier AND heater.

You have a solution to everything. So why is the world in such a mess? (I haven't got lost just yet, though I'm working on it).

Partypooper,
Engineer-Poet was trying to point out that their are solutions to most problems. In the case of car heaters/frost/condensation in cold weather, central Canada or Alaska is a good place to go to see the solutions. Interior temperatures don't have to be above 0C as long as you have dry air on windows or electrically heat them. You wear parkas and boots for warmth, as you are going to need them stepping out of car at -40C.
It's good to raise possible problems but only if you are open to at least evaluating potential solutions in good faith.

Granted, and there likely solutions to every problem we face, but unless the solution makes someone rich, it does not normally happen. This is why the world is in such a mess, because making money (and usually quickly) holds above all else. The ICE/oil industry combination are an examples of this and nuclear power too cheap to meter; is it? Wind power is quoted as a saviour but the recognition that it will require vast energy storage will change the equation again and up will go the cost. At the moment this is conveniently missed out of the equation. Denmark is often quoted in support of wind power, but a little digging deeper reveals why. It is surrounded by countries that can absorb or supply power.
To solve our problems we need to appreciate the scale of them. I see little eveidence of that. Just assuming electric vehicles will seamlessly replace the ICE in 22 years, after a 100 years of gradual investment, growth and development of the former, is optimistic in my opinion. Peak oil is a serious problem and there is no guarantee we can carry on with a transport system we have become used to, it may be unwise to do so in fact. That's MO, you are entitled to yours.

http://www.teslamotors.com/images/content/well_to_wheel_energy.gif

"How can you know, with certainty, how efficient one car is versus another? We conducted a “well-to-wheel” accounting for all fuel efficiency and emissions of several types of high-efficiency cars, including an estimate for the Tesla Roadster, based upon performance prototypes.

Here‘s what we found: the Tesla Roadster offers double the efficiency of popular hybrid cars, while generating one-third of the carbon dioxide. Compare the Tesla Roadster against other sports cars and the results get better still: it is six times as efficient and produces one-tenth the pollution, all while achieving the same performance and acceleration."

http://www.teslamotors.com/efficiency/well_to_wheel.php

Hold on. Where can I buy that?

The car, I mean. Stop laughing, my wife also reads TheOildrum!

This is my first post here - I had to register and make my comment. For what it is worth, here are my calculations for Electric Car via wind power, based more on real battery car solutions today and assuming that the percentage of electricity generated on the grid is above 15% renewable sources (including wind) which are prone to natural spikes and lulls in production - starting to requiring background storage (less reliance on fossil fuels) and production smoothing. The key difference in my calculations is that I assume in scenario 1 that the car batteries themselves are not charged directly from mains and the special infrastructure required for handling the spike voltages generated by mass renewable sources of electricity needs to be added to the existing grid systems. I have taken what I believe are realistic values in efficiency. Values I have seen on Grid Capacity Smoothing/Battery Storage range from as low as 70% to as high as 95%, but I have taken a value of 86% efficiency. This value I admit is highly debatable, but I assume it will be less efficient than Grid transmission losses (<0.90). I have assumed no difference in car battery efficiency under low or mains voltage scenarios, this might well change with new technology.

Scenario 1.

ERoEI for wind ~ 20, efficiency factor = 0.95
Grid Capacity Smoothing/Battery Storage for renewables = 0.86
Grid transmission losses = 0.9
Power inversion (transformer) = 0.92
Battery efficiency = 0.97 (based on low voltage storage)
Motor efficiency = 0.86 (based on low voltage motor system)

Combined efficiency = 56.4%

Scenario 2.

ERoEI for wind ~ 20, efficiency factor = 0.95
Grid Capacity Smoothing/Battery Storage for renewables = 0.86
Grid transmission losses = 0.9
Battery efficiency = 0.97 (based on mains voltage storage)
Motor efficiency = 0.92 (based on mains voltage)

Combined efficiency = 65.6 %

Note: I view just highlighting efficiencies as a comparison between engines as highly debatable, as no consideration into such concepts of peak lithium for future battery production, other limited resources, or energy inputs into research and development (entropic considerations) are taken into account. On the Peak Oil stakes I am closer to Matt Savinar/James Kunstler - Defcon 1.5 :-)